Download as pdf or txt
Download as pdf or txt
You are on page 1of 96

1

• 2
• 3
• 4 A 63-year-old man with schizoaftective disorder is brought to the hospital by his heaHh aide because of a 3-day history of severe
• 5 weakness in his legs, an unsteady gait, and diarrhea. His hands are shaking and he seems mildly confused. He was recently
• 6 hospitalized because of a manic episode and had been placed on a combination of medications. While in the hospital he was also
• 7 diagnosed with chronic renal insufficiency, hypertension, and obesity. His medications were changed and he was placed on a
• 8 combination of lithium, valproate, and quetiapine. Nifedipine has been added to his antihypertensive medication. Which of the

• 9 following statements is correct?

• 10
• 1i
0 A. Blood valproate level should be ordered
• 12
• 13 0 B. Check to see if the patient has been drinking alcohol
• 14 0 C. Condition is due to a gastroenteritis
• 15
• 16 f) 0 D. Condition IS due to drug intoxication
• 17 0 E. The patient will need a dialysis right now
• 18
• 19
• 20 Explanation Rel<aps & Refs 0 Help 0 My Questions
• 21
• 22 The correct answer is 0 . This patient has been diagnosed with chronic renal insufficiency, 82% of students got this correct
• 23 and the addition of nifedipine to antihypertensive medication, along with the prior diagnosis, may
• 24 Send us your feedback at:
have caused an increase in lithium to toxic levels. The symptoms that the patient displays medFeedback@kaplan.com
• 25 (weakness, unsteady gait, diarrhea, tremor, and confusion) are consistent with lithium
Please include QIO s2s148m
• 26 intoxication, so the first thing to do in this case is to obtain the lithium level.
• 27 6 \l'~lnrnQfO lo\IOI ~t\lllA Qlcn ho nhta.ino.rt /,..ht'\.i,...o 6.\ h11f tho C\tmntnrnc QrO lo<:C lilfol\t tn he
• 28
• 29
• 30 Exit
1
• 2
• 3
• 4
• 5
• 6 Explanation ReKaps & Refs f} Help 0 My Questions

• 7
The correct answer i s D . This patient has been diagnosed with chron ic renal insufficiency, 82% of students got this correct.
• 8
9 and the addition of nifedipine to antihypertensive medication, along with the prior diagnosis, may
• Send us your feedback at:
have caused an increase in lithium to toxic levels. The symptoms that the patient displays
• 10 medFeedback@kaplan.com
(weakness, unsteady gait, diarrhea, tremor, and confusion) are consistent with lithium
• 1i Please include QID s2sl48m
intoxication, so the first thing to do in this case is to obtain the lithium level.
• 12
• 13 A valproate level could also be obtained (choice A) , but the symptoms are less likely to be
• 14 caused by its increase. Valproate is metabolized through the liver, not the kidney; chronic renal
• 15 insufficiency should not affect it, nor does nifedipine affect its level. The question asks for you
• 16 to pick the initial step; of the given choices, the best answer is to check lithium levels.
• 17 The patient may be checked for alcohol abuse (choice B) if there is a suspicion of that, but
• 18 only after more urgent and life-threatening causes are ruled out This patient's history gives no
• 19 evidence of alcohol abuse; however, that should always be checked .
• 20
The condition is not due to gastroenteritis (choice C). This man's diarrhea is a manifestation
• 21
of lithium toxicity.
• 22
• 23 The patient may not need a dialysis right now (choice E). The decision of dialysis is based on
• 24 the blood level of lithium, which need to be determined first If the blood level is more than 2,
• 25 dialysis should be started.
• 26
• 27
• 28
• 29
• 30
• 4 An 18-year-old girl is brought to the clinic because she periodically pulls her hair and puts it in her mouth. She has several bald
• 5 patches on both sides of her head. Initially the mother thought it was some sort of infection but the dermatologist "did not find
• 6 anything." The patient describes a sense of relief when she does it and claims that she really does it when upset, anxious. or
• 7 overwhelmed. She admits that she has been doing it on and off for at least 2 years and thinks that it started after her parents got
• 8 divorced. She is doing well in school. Which of the following is the most appropriate initial step in management?

• 9
• 10
0 A. Prescribe a benzodiazepine to relieve tension
• 11
• 12 0 B. Prescribe a tricyclic antidepressant
• 13
(I 0 C. Rule out the presence of comorbid conditions
• 14
• 15 0 D. Start interpersonal psychotherapy
• 16 0 E. Start treatment with carbamazepine
• 17
• 18
• 19 Explanation Rel<aps & Refs 6 Help Q My Questions
• 20
• 21 The correct answer is C . When it is suspected that a patient has trichotillomania, the most 54% of students got this correct
• 22 appropriate initial step is to rule out the presence of comorbid conditions. The most frequent
• 23 Send us your feedback at:
ones would be obsessive-compulsive, borderline, and depressive disorders. medFeedback@kaplan .com
• 24
Prescribing a benzodiazepine (choice A) does not address the problem and may only Please include QIO s2s 158
• 25
episodically relieve symptoms of anxiety. The potential for addiction is greater than the potential
• 26
benefit.
• 27
• 28
• 29
• 30

0 C. Rule out the presence of comorbid conditions
• 4
• 5 0 D. Start interpersonal psychotherapy
• 6
0 E. Start treatment with carbamazepine
• 7
• 8
• 9
• 10
Explanation ReKaps & Refs 6 Help 0 My Questions

• 11
The correct answer is C . When it is suspected that a patient has trichotillomania, the most 54% of students got this correct.
• 12
appropriate initial step is to rule out the presence of comorbid conditions. The most frequent
• 13 Send us your feedback at:
ones would be obsessive-compulsive, borderline, and depressive disorders. medFeedback@kaplan.com
• 14
• 15 Prescribing a benzodiazepine (choice A) does not address the problem and may only Please include QID s2s158

• 16 episodically relieve symptoms of anxiety. The potential for addiction is greater than the potential
• 17 benefit
• 18 Prescribing a tricyclic antidepressant (choice B) is not the appropriate next step. Comorbid
• 19 conditions need to be ruled out first Tricyclics have many side effects and more appropriate
• 20 treatment with SSRI might be available.
• 21
Interpersonal therapy (choice D) is not an appropriate choice for impulse-control disorder.
• 22
Some forms of behavior therapy are more appropriate.
• 23
• 24 Treatment with carbamazepine (choice E) should be considered for intermittent explosive
• 25 disorder, but it is usually not used for trichotillomania.

• 26
• 27
• 28
• 29
• 30
• 1
• 2
• A 36-year-old married woman who works in a nursing home complains of chest pain and pressure. She has had similar episodes in
• 4 the past, and evaluations were normal. Since puberty, she suffered from dysmenorrhea. nausea, vomiting, and fatigue, and she finally
• 5 had a hysterectomy with bilateral salpingo-oophorectomy 5 years ago because of severe menorrhagia and since then complains of
• 6 painful intercourse. She suffers from a "weak stomach." Last year she had an episode of dizziness and unstable gait that lasted for
• 7 "some time"; however, a workup done at the time was negative. She suffers from migraine headaches, and the standard migraine
• 8 medications do not help her and all headache workups have been negative. She does not even bother going to a regular physician for
• 9 her back pain but she sees a chiropractor on a regular basis, which brings some relief.
• 10
• 11
• 12 Which of the following is the most likely diagnosis?
• 13
• 14 O A. Conversion disorder
• 15 0 B. Generalized anxiely disorder
• 16
• 17 0 c. Masked depression

• 18 O D. Pain disorder
• 19
(I 0 E. Somatization disorder
• 20
• 21
• 22
• 23
Explanation ReKaps & Refs 8 Help Q My Questions

• 24
The correct answer is E. Somatization disorder is characterized by the following : multiple 41 % of students got this correct.
• 25
medical complaints resulting in significant diagnostic testing and medical interventions, causing
• 26 Send us your feedback a t:
impaired social and occupational functioning. The symptoms cannot be explained by medical
• 27 medFeedback@kaplan.com
finrlinn~ ThA ~vmntnm~ in,-,lurlP. n::~in in ::~1 IP.::~~t 4 ~iiP.~ nnP. n~P.IIrlnnP.urnlnni,-, ~vmntnm nnA
• 28
• 29
• 30 Exit
• 1
• 2

• 4 The correct answer is E. Somatization disorder is characterized by the following : multiple 41 % of students got this correct
• 5 medical complaints resulting in significant diagnostic testing and medical interventions, causing
Send us your feedback a t:
• 6 impaired social and occupational functioning. The symptoms cannot be explained by medical
medFeedback@kaplan.com
• 7 findings. The symptoms include pain in at least 4 sites, one pseudoneurologic symptom, one
Please include QID s2s181m
• 8 symptom of sexual dysfunction, and at least two or more gastrointestinal symptoms. These
• 9 symptoms are not conciously created by the patient.
• 10 Conversion disorder (choice A) is a constellation of one or more neurologic symptoms
• 11 associated with psychological conflict. The symptoms include deficits affecting motor or sensory
• 12 function under voluntary control, disturbances of consciousness, or pseudoseizures. There is a
• 13 clear temporal association of the onset of symptoms and a stressor. The symptoms are not
• 14 consciously produced and are not caused by any other medical or psychiatric condition .
• 15
Generalized anxiety disorder (choice B) is defined by unrealistic or excessive worry about
• 16
activities or life events that lasts at least 6 months. In addition, the following symptoms must be
• 17
present: fatigability, muscle tension, irritability, trouble falling or staying asleep, difficulties
• 18
concentrating, or restlessness. The symptoms cause significant impairment in everyday
• 19 functioning .
• 20
21 Masked depression (choice C) can be presented through somatic complaints such as

22 cardiovascular, gastrointestinal, urinary, or orthopedic , together with depressive symptoms. The

disorder meets criteria for depressive disorder but usually not for somatization disorder.
• 23
• 24 Pain disorder (choice D) is characterized by pain as a prominent feature together with
• 25 psychologic factors that precipitate, exacerbate, and contribute to its severity. It results in
• 26 impairment of everyday functioning and is not caused by any other psychiatric or medical
• 27 condition
• 28
• 29
• 30 Exit
• 1
• 2
• 3
Over the past several months, a 48-year-old single woman has had a significant change in behavior. She started accusing her sister
of poisoning her to get the entire piece of property they were going to split as an inheritance. She then began accusing her sister,
• 5
who works in a government agency, of plotting against her and getting government officials involved. She started writing numerous
• 6
accusatory letters to government officials and finally hired a private detective to "investigate and collect the evidence." In the
• 7
meantime, she has been working as a personal assistant in a law firm. Mental status examination reveals organized thoughts, an
• 8
appropriate affect, and no hallucinations. She has no personal or family history of medical or psychiatric disorders. Which of the
• 9
following is the most likely diagnosis?
• 10
• 11
• 12 0 A. Brief psychotic disorder
• 13
• 14
0 0 B. Delusional disorder

• 15 0 C. Paranoid personality disorder


• 16
0 D. Schizophrenia paranoid type
• 17
• 18 0 E. Schizophreniform disorder
• 19
• 20
• 21 Explanation Rel<aps & Refs 8 Help Q My Questions

• 22
• 23 The correct answer is B. Delusional disorder is a psychotic disorder with nonbizarre beliefs 45% of students got this correct

• 24 lasting at least 1 month. Apart from the impact of the delusions, functioning is usually not
Send us your feedback a t:
• 25 impaired in other areas of life. Tactile or olfactory hallucinations may be present. The medFeedback@kaplan.com
26 disturbance is not caused by abuse of substances or a general medical condition.
• Please include QID s2s176m
• 27 Brief psychotic disorder (choice A) requires one or more of the following characteristics to be
• 28
• 29
• 30 Exit
• 1

• 5 Explanation ReKaps & Refs @ Help 0 My Questions

• 6
• 7 The correct answer is B. Delusional disorder is a psychotic disorder with nonbizarre beliefs 45% of students got this correct.

• 8 lasting at least 1 month. Apart from the impact of the delusions, functioning is usually not
Send us your feedback at:
impaired in other areas of life. Tactile or olfactory hallucinations may be present. The
• 9 medFeedback@kaplan.com
disturbance is not caused by abuse of substances or a general medical condition.
• 10 Please include QID s2sl76m
• 11 Brief psychotic disorder (choice A) requires one or more of the following characteristics to be
• 12 present: delusions, hallucinations, or disorganized speech. Duration of an episode is from 1 day
• 13 up to 1 month, and it is not caused by any other medical condition or substance abuse.
• 14 Paranoid personality disorder (choice C) is a peiVasive distrust and suspiciousness of others
• 15 that begins in ear1y adulthood and presents with 4 or more of the following characteristics: the
• 16 person is reluctant to confide in others, suspects others without sufficient evidence, is
• 17 preoccupied with doubts about loyalty of friends, sees hidden or threatening meaning in benign
• 18 events, perceives attacks on him/herselfthat are not evident to others, is suspicious of partner
• 19 or spouse, and is unforgiving of insults or injuries The symptoms described in the stem are for
• 20 a few months only, whereas for the diagnosis of paranoid personality disorder, the symptoms
• 21 would have to be present from early childhood
• 22
Schizophrenia (choice D) is diagnosed by the presence of two or more of the following
• 23 characteristics: delusions, hallucinations, disorganized speech or behavior, and negative
• 24 symptoms lasting at least 6 months with at least 1 month of active symptoms. Symptoms cause
• 25 significant impairment in everyday functioning.
• 26
To be diagnosed with schizophreniform disorder (choice E) • symptoms must meet the criteria
• 27
• 28
• 29
• 30 Exit


1
a Item: 4 of 44 <J
Previous
C>
Next
il
Lab Values
,...
Notes Calculator
0111:11 IJ~Y\;110111; 01~01 01:11 \COOICe A) I t14UII 1:1~ 0111:1 01 111011:1 0 1 Ultl IOIIOWIIIY \;II (II Cl\;ltlll~ll\;~ LO Uti
present: delusions, hallucinations, or disorganized speech. Duration of an episode is from 1 day
• 5 up to 1 month, and it is not caused by any other medical condition or substance abuse.
• 6
Paranoid personality disorder (choice C) is a pervasive distrust and suspiciousness of others
• 7 that begins in early adulthood and presents with 4 or more of the following characteristics: the
• 8 person is reluctant to confide in others, suspects others without sufficient evidence, is
• 9 preoccupied with doubts about loyalty of friends, sees hidden or threatening meaning in benign
• 10 events, perceives attacks on him/herself that are not evident to others, is suspicious of partner
• 11 or spouse, and is unforgMng of insults or injuries. The symptoms described in the stem are for
• 12 a few months only, whereas for the diagnosis of paranoid personality disorder, the symptoms
• 13 would have to be present from early childhood
• 14
Schizophrenia (choice D) is diagnosed by the presence of two or more of the following
• 15
characteristics: delusions, hallucinations, disorganized speech or behavior, and negative
• 16 symptoms lasting at least 6 months with at least 1 month of active symptoms. Symptoms cause
• 17 significant impairment in everyday functioning .
• 18
To be diagnosed with schizophreniform disorder (choice E) , symptoms must meet the criteria
• 19
for schizophrenia, which include at least two or more of the following: delusions, hallucinations,
• 20
disorganized speech, catatonic behavior, and negative symptoms. The episode, including
• 21
prodromal, active, and residual symptoms, lasts at least 1 month but <6 months. Exclusion
• 22
criteria are schizoaffective and mood disorders, and conditions related to substance abuse or
• 23
general medical condition.
• 24
• 25 Reviewed on 12/10112
• 26
• 27
• 28
• 29
• 30
• 1
• 2
• 3 A 45-year-old woman with type I bipolar disorder comes to the physician because of worsening of her depressive symptoms and
• 4 weight gain. She had been stable for a while but lately has started to feel tired and slow. Her skin has been drier than usual and her
• 5 husband has noticed a change in her voice. She is concerned that she cannot tolerate stress any more, because her hair is falling
• 6 out. After talking to her and reviewing her medications, the physician runs some tests. When the results return , the physician's
• 7 suspicion is confirmed. Which of the following caused this patient to have the described symptoms?
• 8
• 9
• 10 0 A. Anemia
• 11 0 B. Idiopathic
• 12
0 C. Iodine deficiency
• 13
• 14 (I 0 D. Lrth1um-mduced
• 15
0 E. Valproate-induced
• 16
• 17
• 18
• 19
Explanation Rel<aps & Refs 0 Help Q My Questions

• 20
The correct answer is D . This patient seems to have developed hypothyroidism, which is a 78% of students got th is correct.
• 21
side effect of lithium therapy. Symptoms of lith ium toxic ity consist of diarrhea , severe tremor.
• 22 Send us your feedback at:
polyuria, ataxia, confusion, and seizures, but less serious adverse effects include gastric medFeedback@kaplan.com
• 23
distress, weight gain, tremor, fatigue, and cog nitive deficits. Long-term treatment can result in
• 24 hypothyroidism. Other side effects involve the kidneys, heart, and skin.
Please include QIO s2s 179m
• 25
26 Anemia (c hoice A) can make a patient feel tired but does not explain the other symptoms.

• 27 Idiopathic (ch oice B) is not the answer because the cause is known: her symptoms are due to
• 28
• 29
• 30 Exit
• 1
• 2
• 3
• 4
• 5
• 6 Explanation ReKaps & Refs t} Help Q My Questions

• 7
• 8 The correct answer is D . This patient seems to have developed hypothyroidism, which is a 78% of students got this correct.

• 9 side effect of lithium therapy. Symptoms of lithium toxicity consist of diarrhea, severe tremor,
Send us your feedback at:
10 polyuria, ataxia, confusion, and seizures, but less serious adverse effects include gastric
• medFeedback@kaplan.com
distress, weight gain, tremor, fatigue, and cognitive deficits. Long-term treatment can result in
• 11 Please include QID s2s179m
hypothyroidism. Other side effects involve the kidneys, heart, and skin.
• 12
• 13 Anemia (choice A) can make a patient feel tired but does not explain the other symptoms.
• 14
Idiopathic (choice B) is not the answer because the cause is known: her symptoms are due to
• 15
lithium.
• 16
• 17 Iodine deficiency (choice D) can lead to hypothyroidism but is very rare in the United States.

• 18 This patient has bipolar disorder and probably is on lithium, which must have caused the
hypothyroidism.
• 19
• 20 Valproic acid (choice E) is an anticonvulsant medication that has also been approved by the
• 21 Federal Drug Administration for treatment of manic episodes Its common side effects include
• 22 nausea, sedation, weight gain, transitory hair loss, and transitory increase in liver function
• 23 tests. Rare side effects are related to hepatitis, pancreatitis, and possible decrease in platelets
• 24 or platelet dysfunction. This patient's symptoms are more consistent with hypothyroidism, which
• 25 is associated with lithium.
• 26
• 27
• 28
• 29
• 30
• 1
• 2
• 3
4 A 59-year-old African American woman, who has been HIV-positive for the past 12 years, is brought to the emergency department

following an episode of combativeness at home. Her family reports that over the past couple of days she has been talking "out of her
• 5
head," stating that her daughter-in-law has been putting poison into the food . She is admitted but refuses to eat or take medication in

• 7 the hospital, stating that the physicians are also against her. Her last CD4 count, done 6 months ago, was 85/mm 3 . Physical and

• 8 neurologic examinations are unremarkable and nonfocal. The current workup for HIV-related infection is negative, and laboratory tests
• 9 are otherwise unremarkable. On Folstein's Mini Mental Status Examination she scores 21 /30, missing on delayed recall, three-stage
• 10 command, writing, and copying, as well as orientation to year. Which of the following is the most likely diagnosis?

• 11
• 12
O A.Amok
• 13
• 14 0 B. Delirium
• 15 0 C. Delusional disorder
• 16
• 17 (I 0 0 . Dementia
• 18 0 E. Schizophrenia
• 19
• 20
• 21 Explanation Rel<aps & Refs 8 Help Q My Questions
• 22
• 23 The correct answer is D. HIV dementia is characterized by affective, cognitive, behavioral, 69% of students got this correct.
• 24 and motor symptoms and signs. It presents as a subcortical process and is most likely to occur
• 25 Send us your feedback at:
in patients with a C04 count below 200/mm 3 . It usually has a slow onset, and after a period of medFeedback@kaplan.com
• 26
stability there can be a precipitous decline. The diagnosis is made when other causes of Please include QID Q0458m
• 27 r1olirinm "ro ov~lut1c.t1 · th~ rti~o~~o m~v nro~ont with nc:vf"hnc::.ic::. within 1-11\/ r1omonti" Tho
• 28
• 29
• 30 Exit
• 1
• 2
• 3
• 4
• 5 Explanation ReKaps & Refs ~ Help Q My Questions

• 7 The correct answer is D. HIV dementia is characterized by affective, cognitive, behavioral, 69'1. of students got this correct.
• 8 and motor symptoms and signs. It presents as a subcortical process and is most likely to occur
Send us your feedback at:
• 9 in patients with a CD4 count below 200/mm 3 . It usually has a slow onset, and after a period of medFeedback@kaplan.com
• 10 stability there can be a precipitous decline. The diagnosis is made when other causes of Please include QID Q0458m
• 11 delirium are excluded; the disease may present with psychosis within HIV dementia. The
• 12 symptoms are controlled with low doses of neuroleptics.
• 13
Amok (choice A) is a culture-bound syndrome. It is seen among Malayans and presents as
• 14
unprovoked outbursts of wild rage , causing the person to run madly and attack others or commit
• 15
suicide. There is a period of amnesia afterward.
• 16
• 17 Delirium (choice B) is the most common cause of mental status changes in patients with
• 18 HIV/AIDS. It may be related to the primary effects of HIV infection in the CNS, the secondary
• 19 effects of systemic HIV disease, or side effects of medications. It may also be a result of other
• 20 usual causes of delirium (e.g., endocrine, metabolic , seizures, trauma, neoplasms). Typically,
21 the clinical picture includes a waxing and waning level of consciousness and fl uctuations in

cognitive functions, as seen in delirium associated with other causes.
• 22
• 23 Delusional disorder (choice C) is defined as a psychotic disorder with persistent nonbizarre
• 24 delusions that are firmly held and may be, for example, of persecution, somatic nature,
• 25 grandeur, or jealousy.
• 26
Schizophrenia (choice E) is a psychosis with the onset of symptoms typically in young
• 27 adulthood. Patients with schizoohrenia can develoo dementia and. if thev are HIV-oositive. also
• 28
• 29
• 30 Exit
• 1
• 2
• 3 Send us your feedback at:
• 4 in patients with a CD4 count below 200/mm 3 . It usually has a slow onset, and after a period of medFeedback@kaplan.com

• 5 stability there can be a precipitous decline. The diagnosis is made when other causes of Please include QID Q0458m
delirium are excluded; the disease may present with psychosis within HIV dementia. The
• 7 symptoms are controlled with low doses of neuroleptics.
• 8 Amok (choice A) is a culture-bound syndrome. It is seen among Malayans and presents as
• 9 unprovoked outbursts of wild rage, causing the person to run madly and attack others or commit
• 10 suicide. There is a period of amnesia afterward.
• 11
Delirium (choice B) is the most common cause of mental status changes in patients with
• 12
HIV/AIDS. It may be related to the primary effects of HIV infection in the CNS, the secondary
• 13
effects of systemic HIV disease, or side effects of medications. It may also be a result of other
• 14
usual causes of delirium (e.g ., endocrine, metabolic, seizures, trauma, neoplasms). Typically,
• 15
the clinical picture includes a waxing and waning level of consciousness and fluctuations in
• 16 cognitive functions, as seen in delirium associated with other causes.
• 17
• 18 Delusional disorder (choice C) is defined as a psychotic disorder with persistent nonbizarre

19 delusions that are firmly held and may be, for example, of persecution, somatic nature,

grandeur, or jealousy.
• 20
• 21 Schizophrenia (choice E) is a psychosis with the onset of symptoms typically in young
• 22 adulthood. Patients with schizophrenia can develop dementia and, if they are HIV-positive, also
• 23 develop symptoms related to HIV. Typical schizophrenic symptoms include delusions and
• 24 hallucinations of at least 1 month's duration and a significant duration of illness and impairment
• 25 of more than 6 months.

• 26
• 27
• 28
• 29
• 30
• 1
• 2
• 3
A 26-year-old man comes to his primary care physician complaining that he has been more and more afraid to leave his house
• 4
because he has been having frequent episodes of sudden discomfort without warning , in which he develops a fear of losing control.
• 5
He is worried that he will have an attack at some time while he is in public. He has no other medical conditions. Physical examination
• 6 is unremarkable. Which of the following is the most appropriate initial medication for this patient in an acute situation?
• 7
• 8
• 9 0 A. Buspirone
• 10
0 B. Chlorpromazine
• 11
• 12 fl 0 C. Clonazepam
• 13
0 0 . Mirtazapine
• 14
• 15 0 E. Trifluoperazine

• 16
• 17
• 18 Explanation Rel<aps & Refs fj Help Q My Questions

• 19
• 20 The correct answer is C. This patient's symptoms (frequent episodes of sudden discomfort 60% of students got this correct.

• 21 without warning, in which he develops a fea r of losing control) suggest that he has panic
Send us your feedback at:
• 22 disorder, which is most appropriately treated with medFeedback@kaplan.com
• 23 • A benzodiazepine (acutely) with a medium-length half-life and duration of action. such as Please include QIO Q0394m
• 24 clonazepam. lorazepam, or alprazolam, which is then tapered off; it works within minutes
• 25 • An antidepressant (long-term) such as an SSRI, TCA, or MAO-I for at least 6 months.
• 26
Psychotherapeutic interventions to treat panic disorder include relaxation techniques and
• 27 <:\lc;:tornil" r4oc:onc:iti"7atinn
• 28
• 29
• 30 Exit
• 1
• 2
• 3
• 4
0 E. Trifluoperazine
• 5
• 6
• 7
• 8
Explanation ReKaps & Refs tj Help Q My Questions

• 9
The c orrect answer i s C. This patienrs symptoms (frequent episodes of sudden discomfort 60% of students got this correct
• 10
without warning, in which he develops a fear of losing control) suggest that he has panic
• 11 Sen d us your feedback at:
disorder, which is most appropriately treated with: medFeedback@kaplan.com
• 12
• 13 • A benzodiazepine (acutely} with a medium-length half-life and duration of action, such as Please include QID Q0394m
• 14 clonazepam, lorazepam, or alprazolam, which is then tapered off; it wor1<s within minutes
• 15 • An antidepressant (long-term) such as an SSRI, TCA, or MAO-I for at least 6 months.
• 16 Psychotherapeutic inteiVentions to treat panic disorder include relaxation techniques and
• 17 systemic desensitization.
• 18
Buspirone (choice A) is an anxiolytic medication that has not been shown to be effective in the
• 19
treatment of panic attacks. It is used in generalized anxiety disorder, where patients have
• 20
excessive and poorly controlled anxiety about life c ircumstances that continues for more than 6
• 21
months.
• 22
• 23 Chlorpromazine (choice B) and trifluoperazine (choice E) are antipsychotics that are not
• 24 indicated for treatment of panic attacks. Mirtazapine (choice D) is an antidepressant that is
• 25 not typically indicated for treatment of panic attacks.

• 26
• 27
• 28
• 29
• 30
• 1
• 2
• 3
A 36-year-old single African American woman is referred for psychiatric evaluation by her internist. She reports a history of
• 4
abdominal pain for the past 9 months after the death of her mother. On interview, the abdominal pain is described as intermittent and
• 5
sometimes related to meals. She also complains of nausea, bloating, and frequent gas. She denies any changes in her bowel
• 6
movements, blood in her stool, or significant weight change in the past few months. She has consulted with multiple specialists,
• 7
including her internist, a gastroenterologist, and a gastrointestinal surgeon. She has had multiple workups, including a Gl series,
radiographs, endoscopy, and colonoscopy that have all been within normal limits. The patient is quite anxious, frustrated, and
• 9 concerned that she is really sick and that her doctors have not been careful enough thus far to find out what is really wrong with her.
• 10 What is the most likely diagnosis?
• 11
• 12
• 13 0 A. Body dysmorphic disorder
• 14
0 B. Conversion disorder
• 15
• 16 (I 0 C. Hypochondriasis
• 17 0 D. Pain disorder
• 18
• 19 0 E. Somatization disorder
• 20
• 21
• 22 Explanation ReKaps & Refs 6 Help Q My Questions

• 23
• 24 The correct answer is C. Hypochondriasis is a fea r of disease and a preoccupation with the 57% of students got this correct.
body that manifests through multiple somatic complaints. This patient has persistent fears of
• 25 Send us your feedback at:
having a Gl illness despite consultations with multiple medical specialists who have done
• 26 medFeedback@kaplan.com
workups that are negative. The duration of the symptoms in hypochondriasis has to be at least 6
• 27 Please includE - --
• 28
• 29
• 30 Exit
• 1
• 2
• 3
• 4 Explanation Rel<aps & Refs 0 Help 0 My Questions

• 5
The correct answer is C. Hypochondriasis is a fear of disease and a preoccupation with the 57% of students got this correct
• 6
body that manifests th rough multiple somatic complaints. This patient has persistent fea rs of
• 7 Send us your feedback at:
having a Gl illness despite consultations with multiple medical specialists who have done
medFeedback@kaplan.com
workups that are negative. The duration of the symptoms in hypochondriasis has to be at least 6
• 9 Please include QID m001057
months. Doctor-shopping and stress in the physician-patient relationship are common.
• 10
Psychosocial stressors include serious illness of a family member or the death of someone
• 11
close to the individual; these can precipitate hypochondriasis in some cases.
• 12
• 13 Body dysmorphic disorder (choice A) is defined as a preoccupation with an imagined defect

• 14 in one's appearance. It is usually restricted to concern about one's appearance rather than a

15 worry about being sick or having an illness.



• 16 Conversion disorder (choice B) is defined as the development of unexplained neurologic or
• 17 general medical symptoms in the context of a strong emotional stressor.
• 18
Pain disorder (choice D) is defined as pain in one or more anatomic sites that causes
• 19
clinically significant distress. Psychologic factors are believed to play an important role in the
• 20
onset and severity of the symptoms, but they are not intentionally produced or feigned .
• 21
• 22 Somatization disorder (choice E) is defined as multiple physical complaints (including four
• 23 pain symptoms, two gastrointestinal symptoms, one sexual symptom, and one neurologic

24 symptom) that after appropriate medical investigation cannot be fully explained or the

impairment is in excess of what would be expected from the history, physical, or laboratory
• 25
findings.
• 26
• 27
• 28
• 29
• 30
• 1
• 2
• 3 A 30-year-old woman comes to her primary care physic ian with multiple symptoms. She complains of feeling sad and anxious for
• 4 about 2 months. She describes difficulty falling asleep at night, decreased appetite with a 15-lb weight loss, diminished interest in her
• 5 hobbies, and poor concentration. The patient denies any other problems. Physical examination is within normal limits. The patient and
• 6 her physician discuss initiating treatment with a selective serotonin reuptake inhibitor (SSRI). Which of the following would be the
• 7 most important additional information to gather before starting the SSRI?
• 8
• 9
• 10 0 A. Family history of alcoholism or drug dependence
• 11 0 B. Family history of depressive episodes
• 12
• 13 fl 0 C. Past h1story of hypomanic or manic episodes

• 14 0 0 . Past history of kidney diseases


• 15
0 E. Past history of obsessions or compulsions
• 16
• 17
• 18
• 19
Explanation Rel<aps & Refs 0 Help 0 My Questions

• 20
The correct answer is C. Although all of the choices are valuable pieces of further 66% of students got th is correct.
• 21
info rmation, a past history of hypomanic or manic episodes is the one choice that might
• 22 Send us your feedback at:
significantly alter treatment planning. It is likely that all antidepressants, including SSRis, have a
• 23 medFeedback@kaplan .com
potential to cause a switch into hypomania or mania, or to accelerate cycling in patients with
• 24 Please include QIO Q0373m
bipolar disorder. If the patient has a history of hypomanic or manic episodes, her depression
• 25 would be considered a bipolar rather than a unipolar depression. In that case, the patient would
• 26 likely need to be started on a mood stabilizer before beginning treatment with an antidepressant
• 27 medication.
• 28
• 29
• 30 Exit
• 1
• 2
• 3
• 4
• 5 The correct answer is C. Although all of the choices are valuable pieces of further 66% of students got this correct

• 6 information, a past history of hypomanic or manic episodes is the one choice that might
Send us your feedback at:
7 significantly alter treatment planning. It is likely that all antidepressants, including SSRis, have a
• medFeedback@kaplan.com
potential to cause a switch into hypomania or mania, or to accelerate cycling in patients with
• 8 Please include QID Q0373m
bipolar disorder. If the patient has a history of hypomanic or manic episodes, her depression
• 9
would be considered a bipolar rather than a unipolar depression. In that case, the patient would
• 10
likely need to be started on a mood stabilizer before beginning treatment with an antidepressant
• 11
medication.
• 12
• 13 A family history of alcoholism or drug dependence (choice A) is also useful information,
• 14 though it would probably not alter the treatment plan. A family history of addictive disorders
• 15 increases the likelihood of the patient experiencing primary mood disorders or substance-use
16 disorders. SSRis are not addictive.

• 17 A family history of depressive episodes (choice B) is useful information in supporting the
• 18 diagnosis of a primary mood disorder. This detail would probably not significantly alter treatment
• 19 planning, although a clear response to one antidepressant in a family member might support the
• 20 use of that same medication in the patient.
• 21
A past history of kidney disease (choice D) is relevant in any patient but is not especially
• 22 important in this case. SSRis are primarily metabolized through the liver's cytochrome P450
• 23 microsomal enzymes.
• 24
25 A past history of obsessions or compulsions (choice E) is included in gathering any

psychiatric history but is not of particular importance in this case.
• 26
• 27 Reviewed on 1216112
• 28
• 29
• 30 Exit
• 1
• 2
• 3
4 An 18-year-old woman comes to her doctor along with her mother, who states that she doesn't know what to do with her daughter.

Ever since the daughter started modeling, she became concerned about being fat. The daughter states that she has a fear of gaining
• 5
weight, for which she binges and purges. She had not had her menstrual period for 5 months. She denies use of laxatives or diuretics.
• 6
On examination, she is thin and has fine hair all over her body. She has scars on her knuckles and poor dentition. Her blood pressure
• 7
is 90/70 mm Hg, pulse is 54/min and regular, and BMI is 18. Which of the following is the most appropriate next step in management?
• 8
• 9
(I 0 A. Adm1t the patient to the hospital to reestablish weight and correct abnonnahlies
• 11
12
0 B. Refer the patient for psychodynamic psychotherapy

• 13 0 C. Start an SSRI
• 14
0 D. Start the patient on honnone therapy to induce her menstrual cycle
• 15
• 16 0 E. Tell the mother not to worry and send them both home
• 17
• 18
• 19 Explanation Rel<aps & Refs f) Help 0 My Questions
• 20
• 21 The correct answer is A. The diagnosis is anorexia because: 73% of students got th is correct.

• 22
• The patient is grossly underweight Send us your feedback at :
• 23 • The patient has manifestations of hormonal imbalance such as amenorrhea, osteoporosis, medFeedback@kaplan.com
• 24 hair loss Please include QIO Q0782m
• 25 • The patient has intense fear of gaining weight
• 26
Immediate hospitalization is indicated when life-threatening or potentially lethal abnormalities are
• 27 ~............................. - : ........ : ....... I 1........... :6 .... 1:-.-.6: ...... ;,.. ............ _ .... ,.. .... _.. ................ ;..,. ........... - .... .,;..., •&... ....... ; ........ t:- :.... • ...... : ....... : ....... ....
• 28
• 29
• 30 Exit
• 1
• 2
• 3
• 4
• 5 The correct answer is A. The diagnosis is anorexia because: 73% of students got this correct

• 6 • The patient is grossly underweight Send us your feedback a t:


• 7 • The patient has manifestations of hormonal imbalance such as amenorrhea, osteoporosis, medFeedback@kaplan.com
• 8 hair loss Please include QID Q0782m
• 9 • The patient has intense fear of gaining weight

Immediate hospitalization is indicated when life-threatening or potentially lethal abnormalities are


• 11
found on examination. Hospitalization is more common in anorexia than in bulimia. Indications
• 12
for hospitalization include the following:
• 13
• 14 • Bradycardia or other rhythm disturbance
• 15 • Severe electrolyte abnormalities, especially of potassium, sodium, and phosphorus levels
• Altered mental status or suicidal ideation
• 16
• Extremely low body weight (BMI <18.5 means grossly underweight)
• 17
• 18 This patient has significant weight loss and bradycardia She may need a full evaluation of her
• 19 physical status to determine and correct abnormalities because her refusal to eat has placed
• 20 her in a potentially life-endangering situation.
• 21 Even though the patient may have psychological issues, psychodynamic psychotherapy
• 22 (choice B) has not proven to be the therapy of choice for anorexic patients.
• 23
Starting an SSRI (choice C) is the treatment for bulimia. Bulimia is the diagnosis, rather than
• 24
anorexia, whenever:
• 25
• 26 • The patient's weight is normal or slightly above average
• 27 • The patient has no manifestations of hormonal imbalance such as amenorrhea,
• 28
• 29
• 30 Exit
• 1
• 2
• 3
• 4 • Bradycardia or other rhythm disturbance
• 5 • Severe electrolyte abnormalities, especially of potassium, sodium, and phosphorus levels
• 6 • Altered mental status or suicidal ideation
• 7 • Extremely low body weight (BMI <18.5 means grossly underweight)
• 8
This patient has significant weight loss and bradycardia. She may need a full evaluation of her
• 9
physical status to determine and correct abnormalities because her refusal to eat has placed
her in a potentially life-endangering situation.
• 11
• 12 Even though the patient may have psychological issues, psychodynamic psychotherapy
• 13 (choice B) has not proven to be the therapy of choice for anorexic patients.
• 14 Starting an SSRI (choice C) is the treatment for bulimia. Bulimia is the diagnosis, rather than
• 15 anorexia, whenever:
• 16
• The patienfs weight is normal or slightly above average
• 17
• The patient has no manifestations of hormonal imbalance such as amenorrhea,
• 18
osteoporosis, hair loss
• 19
• 20 Starting hormone therapy to induce her menstrual period (choice D) would be inappropriate
• 21 since the evaluation and treatment of the underlying cause of the condition have not been
• 22 addressed.
• 23 Telling the mother not to worry and sending the mother and daughter home (choice E) would
• 24 be a completely inappropriate and inadequate professional assessment of the situation, as this
• 25 patient may be in a life-threatening condition .
• 26
• 27
• 28
• 29
• 30
• 1
• 2
• 3
• 4 A 38-year-old white man is brought to the physician by his mother. The mother states that the patient has become increasingly
• 5 paranoid and irritable over the last 2 years. He suspects her of poisoning his food . He has few friends and has never moved out of the
• 6 family house, in which he grew up. The physician notices the man's random, uncontrollable, jerky movement. The patient scores 24 on
• 7 the Mini-Mental Status Examination. Which of the following would be seen in a head CT scan of this patient?
• 8
• 9
10
0 A. Bilateral multiple infarcts in the dominant hemisphere

• 1.1. (I 0 B. Caudate nucleus atrophy
• 12
0 C. Loss of dopaminergic neurons in the substantia nigra
• 13
• 14 0 0 . Ventricular enlargement and reduction in cortical volume
• 15
• 16
• 17 Explanation ReKaps & Refs 0 Help Q My Questions
• 18
• 19 The co rrect answer is 8 . The diagnosis is Huntington disease, an autosomal-dominant 68% of students got this correct.

• 20 disorder that involves degeneration of neurons in the basal ganglia, producing:


Send us your feedback at:
• 21 • Movement disorder: choreoathetoid movements, fidgetiness, uncontrollable jerkiness medFeedback@kaplan. com
• 22 • Cognitive disorder: dementia (low MMSE scores in this patient) Please include QIO s2s 141m
• 23 • Behavioral disorder: may include depression, psychosis, and personality changes
• 24 (paranoid and irritable attitude)
• 25
CT or MRI of head in a patient with Huntington will show atrophy of the caudate nucleus.
• 26
• 27 A finding of bilateral multiple infarcts in the dominant hemisphere (choice A) on CT is
• 28
• 29
• 30 Exit
• 1
• 2
• 3
• 4 The corr ect answer is B . The diagnosis is Huntington disease, an autosomal-dominant 68% of students got this correct.
• 5 disorder that involves degeneration of neurons in the basal ganglia, producing:
Send us your feedback a t:
• 6
• Movement disorder: choreoathetoid movements, fidgetiness, uncontrollable jerkiness medFeedback@kaplan.com
• 7
• Cognitive disorder: dementia (low MMSE scores in this patient) Please include QID s2sl4lm
• 8
• Behavioral disorder: may include depression, psychosis, and personality changes
• 9
(paranoid and irritable attitude)
• 10
• 11. CT or MRI of head in a patient with Huntington will show atrophy of the caudate nucleus.
• 12 A finding of bilateral muHiple infarcts in the dominant hemisphere (choice A) on CT is
• 13 consistent with a diagnosis of vascular dementia, which would present with a stepwise
• 14 deterioration of cognitive functions, with the motor deficits corresponding to new infarctions .
• 15
loss of dopaminergic neurons in the substantia nigra (choice C) on CT is consistent with a
• 16
diagnosis of Parkinson disease, which would present with resting tremor, rigidity, and
• 17
bradykinesia. This patient does not present with this history
• 18
• 19 Ventricular enlargement and reduction in cortical volume (choice D) on CT are consistent with
• 20 a diagnosis of Alzheimer disease or schizophrenia
• 21 • Alzheimer disease is usually characterized by onset later in life (older than 65 years).
• 22 Patients with Alzheimer disease do not typically present with choreoathetoid movements
• 23 and are usually substantially older than this man.
• 24 • Even though this patient's paranoia sounds like schizophrenia, schizophrenics do not
• 25 present with dementia and movement disorders .
• 26
• 27
• 28
• 29
• 30
• 1
• 2
• 3
• 4 A 63-year-old man is brought to the clinic by his wife, who is concerned that he is in the early stages of senile dementia, Alzheimer
• 5 type. The patient has been having memory disturbances lately and his wife reports that he is "not being as sharp intellectually as he
• 6 used to be." Cognitive testing reveals that the patient has some deficiencies in memory and calculation. To test the patient's abstract
• 7 thinking, wh ich of the following is the most appropriate evaluation?
• 8
• 9
10
0 A. Copy a para

• 11 0 B. Give the current date

f) 0 C. Interpret a proverb
• 13
• 14 0 0 . Name the past frve presidents
• 15 0 E. Count backward from 100 in increments of 7
• 16
• 17
• 18
Explanation ReKaps & Refs @ Help 0 My Questions
• 19
• 20 The correct answer is C . Abstract thinking is defined as the ability to appreciate symbolic or 72"/o of students got this correct.
• 21 metaphoric meaning. Interpretation of a proverb tests this ability. For example, "A rolling stone
• 22 gathers no moss" would be interpreted word-by-word (concrete thinking) by a patient with
Send us your feedback at:
medFeedback@kaplan.com
• 23 impaired abstract thinking .
• 24 Please include QJO s2s 131m
Copying a para (c hoice A) tests a patient's visuospatial ability.
• 25
• 26 Giving the current date (choice B) tests a patient's orientation and is useful in the assessment
• 27 of delirium as well as dementia. II does not involve the use of abstract thinking.
• 28
• 29
• 30 Exit
• 1
• 2
• 3
• 4
5
(I 0 C. Interpret a proverb

• 6 0 D. Name the past five presidents
• 7
0 E. Count backward from 100 in increments of7
• 8
• 9
• 10
Explanation ReKaps & Refs 6 Help 0 My Questions
• 11

The correct answer is C . Abstract thinking is defined as the ability to appreciate symbolic or 72% of students got this correct.
• 13
metaphoric meaning. Interpretation of a proverb tests this ability. For example, "A rolling stone
• 14 gathers no moss" would be interpreted word-by-word (concrete thinking) by a patient with Send us y our feedback at:
medF eedback@kaplan .com
• 15 impaired abstract thinking.
• 16 Please include QID s2s131m

• 17 Copying a para (choice A) tests a patient's visuospatial ability.

• 18 Giving the current date (choice B) tests a patient's orientation and is useful in the assessment
• 19 of delirium as well as dementia. It does not involve the use of abstract thinking .
• 20
Naming the past five presidents tests a patient's fund of knowledge. It is a memory test that does
• 21
not involve the use of abstract thinking Therefore, choice D is incorrect.
• 22
• 23 "Serial sevens" (counting down from 100 in increments of 7; choice E ) is a test of a patient's
• 24 ability to sustain attention to a continued task. It is a test of concentration and not of abstract
• 25 thinking.

• 26
• 27
• 28
• 29
• 30
• 1
• 2
• 3
• 4 A 26-year-old man is brought to the hospital by his family after silting in his room with the lights out and the door closed for two days.

5 He has not eaten over this time. About a week ago, the family noticed the patient becoming increasingly agitated and paranoid about

cars driving by on the street in front of their house. He covered the windows of his bedroom with newspaper and unplugged his radio
• 6
and television. At night, he was heard pacing in his room and talking to himself. Although he had taken olanzapine after a psychiatric
• 7
hospitalization about 6 months previously, the family reported that he threw away the medication about a month ago. On admission,
• 8
the patient is silting in a chair with his head hung low. He is disheveled and malodorous, after having urinated on himself several times
• 9
over the past two days. During physical examination, the patient appears to be awake, but firmly resists any attempts to be moved. He
• 10
does not follow instructions, and the nurse was unable to move his arm to obtain a blood pressure measurement without assistance.
• 11 Which term would best describe this patienrs resistance to being moved?
• 12

• 14 0 A. Akathisia
• 15
0 B. Cataplexy
• 16
• 17 0 C. Echopraxia
• 18 (I 0 D. Negativism
• 19
• 20 0 E. Stereotypy

• 21
• 22
• 23 Explanation ReKaps & Refs @ Help 0 My Questions

• 24
• 25 The correct answer is D. This patient is exhibiting the catatonic symptom known as 51 % of students got this correct

26 negativism, which is a motiveless resistance to all attempts to be moved or to all instructions.


• Send us your feedback at:
Signs of cataton ia include stupor, negativism, rigidity, posturing, mutism, stereotypies,


27
28
- -----=---- ··· - --- ~ - .. -··-:..&- - -- -·
! 1.. ! 1 !.£,.. _ _ _. __ ._. __ ; _ --·-·--=-_____ . . ______ ;_. _. . . .:. . medFeedback""h-• ~- ~~-

• 29
• 30 Exit
• 1
• 2
• 3
• 4
• 5 Explanation ReKaps & Refs 0 Help Q My Questions
• 6
• 7 The correct answer is D. This patient is exhibiting the catatonic symptom known as 51% of students got this correct.
• 8 negativism, which is a motiveless resistance to all attempts to be moved or to all instructions.
9 Send us your feedback at:
• Signs of catatonia include stupor, negativism, rigidity, posturing, mutism, stereotypies,
medFeedback@kaplan.com
• 10 mannerisms, waxy flexibility, and catatonic excitement. Catatonia may be associated with
Please include QID Q0456
• 11 schizophrenia (catatonic type), mood disorders (with catatonic features), or general medical
• 12 conditions.
• Akathisia (choice A) is usually classified as an extrapyramidal side effect of antipsychotic
• 14 medications. It is characterized by a subjective feeling of muscular tension (an inner sense of
• 15 restlessness) that can cause distressing restlessness , pacing, or repeated movements. It may
• 16 be mistaken for psychotic agitation, and thus inappropriately treated.
• 17
Cataplexy (choice B) is a temporary loss of muscle tone and weakness precipitated by a
• 18
variety of emotional states. It is most characteristically associated with narcolepsy.
• 19
• 20 Echopraxia (choice C) is a pathological imitation of the movements of one person by another.
• 21 It can be seen in catatonia, delirium, dementia, and other disorders.
• 22 Stereotypy (choice E) is a repetitive fixed pattern of physical action, movement, or speech. It
• 23 may be seen in catatonia. This patient does not currently exhibit stereotypies.
• 24
• 25 Reviewed on 12/10112
• 26
• 27
• 28
• 29
• 30
• 1
• 2
• 3
• 4 A 37-year-old IV heroin abuser, who is HIV-positive and has many legal and social problems, has been admitted to the hospital for

• 5 treatment of pneumonia. During his stay, he intermittently refuses to undergo procedures or cooperate with doctors but insists on
going out to smoke and having visitors. On one occasion, a visitor was caught injecting drugs into his IV infusion. When confronted,
• 6
the patient throws a tray at the staff and accuses them of being unprofessional. Which of the following is the most likely diagnosis?
• 7
• 8
• 9
(I 0 A. Ant1soc1al personality disorder
• 10
• 11 0 B. Border1ine personality disorder
• 12 0 C. Conduct disorder
• 13
0 0 . Major depressive disorder with anxious/agitated features
• 15 0 E. Paranoid personality disorder
• 16
• 17
• 18 Explanation ReKaps & Refs 0 Help 0 My Questions
• 19
• 20 The correct answer is A. Antisocial personality disorder is defined by a pervasive pattern of 66% of students got this correct.
• 21 disregard for and violation of the rights of others. It occurs after age 15 and is accompanied by
Send us your feedback at:
• 22 at least 3 of the following: failure to conform to social norms or laws as indicated by repeated medFeedback@kaplan.com
• 23 arrests, deceitfulness, irritability and aggressiveness, disregard for safety of others,
Please include QJO Q0078
• 24 irresponsibility, and a lack of remorse after mistreating others. The disorder is not due to
• 25 schizophrenia or manic episode.
• 26 Border1ine personality disorder (choice B) is characterized by a pervasive pattern of instability
• 27 in interpersonal relationships, self-imaQe, and affect. as indicated by 5 or more of the followinQ:
• 28
• 29
• 30 Exit
• 1
• 2
• 3
• 4 Explanation ReKaps & Refs 6 Help 0 My Questions

• 5
• 6 The correct answer is A. Antisocial personality disorder is defined by a pervasive pattern of 66% of students got th is correct

• 7 disregard for and violation of the rights of others. It occurs after age 15 and is accompanied by
Send us your feedback at:
at least 3 of the following : failure to conform to social norms or laws as indicated by repeated
• 8 medFeedback@kaplan.com
arrests, deceitfulness, irrilability and aggressiveness, disregard for safety of others,
• 9 Please include QID Q0078
irresponsibility, and a lack of remorse after mistreating others. The disorder is not due to
• 10
schizophrenia or manic episode.
• 11
• 12 Borderline personality disorder (choice B) is characterized by a pervasive pattern of instability
• 13 in interpersonal relationships, self-image, and affect, as indicated by 5 or more of the following:
impulsivity, effor1s to avoid real or imagined abandonment, unstable relationships, affective
• 15 instability, feelings of emptiness, difficulty controlling anger, and identity disturbance.

• 16 Conduct disorder (choice C) involves a repetitive pattern of behavior in which the rights of
• 17 others or social norms are violated. It must begin in childhood, and at least one criterion has to
• 18 be present for 6 months. Three or more of the following criteria are needed to establish the
• 19 diagnosis: cruelty to animals or people, destruction of property, deceitfulness or theft, and
• 20 serious violations of rules.
• 21
Major depressive disorder (choice D) requires a 2-week period of depressed mood or
• 22
anhedonia, along with associated symptoms of decreased energy, changes in sleep, changes
• 23 in appetite and weight, changes in psychomotor activity, presence of guilt, or suicidal ideation.
• 24 These symptoms are not secondary to another mental disorder or general medical condition.
• 25
Paranoid personality disorder (choice E) is diagnosed when there is a pervasive pattern of
• 26
distrust of others beginning in adulthood and indicated at least by 4 of the following : reads
• 27 • .................... ; ....... _ ............. : ....... : ...................... ; ........... _ .....,.. .......... , _........................... ;,..;...,., .................... ;, ..... ;-l;,.. ..... ;... .... ... ...................... .....
• 28
• 29
• 30 Exit
• 1
• 2
• 3
schizophrenia or manic episode.
• 4
• 5 Borderline personality disorder (choice B) is characterized by a pervasive pattern of instability
• 6 in interpersonal relationships, self-image, and affect, as indicated by 5 or more of the following :
• 7 impulsivity, efforts to avoid real or imagined abandonment, unstable relationships, affective
• 8 instability, feelings of emptiness, difficulty controlling anger, and identity disturbance.
• 9 Conduct disorder (c hoice C) involves a repetitive pattern of behavior in which the rights of
• 10 others or social norms are violated. II must begin in childhood, and at least one criterion has to
• 11 be present for 6 months. Three or more of the following criteria are needed to establish the
• 12 diagnosis: cruelty to animals or people, destruction of property, deceitfulness or theft, and
• 13 serious violations of rules .

Major depressive disorder (choice D) requires a 2-week period of depressed mood or


• 15 anhedonia, along with associated symptoms of decreased energy, changes in sleep, changes
• 16 in appetite and weight, changes in psychomotor activity, presence of gum, or suicidal ideation.
• 17 These symptoms are not secondary to another mental disorder or general medical condition.
• 18
Paranoid personality disorder (choice E) is diagnosed when there is a pervasive pattern of
• 19
distrust of others beginning in adulthood and indicated at least by 4 of the following: reads
• 20
threatening meaning into benign events, recurrently suspicious without justification, reluctant to
• 21
confide in others, persistently bears grudges, is preoccupied with unjustified doubts, and
• 22
suspects others are exploiting harming or deceiving him. The disorder is not due to
• 23
schizophrenia, mood disorder, or a general medical condition.
• 24
• 25 Reviewed on 12/10112
• 26
• 27
• 28
• 29
• 30
• 1
• 2
• 3
During a trip out of town, a 25-year-old woman is involved in a minor motor vehicle accident and is brought to the emergency
• 4
department. She has no injuries, aside from minor abrasions. During the observation period, the physician notices that her vital signs
• 5
are increased and that she seems confused, tremulous, and restless. During the initial intake, the patient reports that the only
• 6
medication she uses is alprazolam, which is prescribed for her to take "as needed" for anxiety attacks. She forgot to go to her doctor
• 7
for it before the trip and she ran out of it a few days ago. She thought she was fine anyway and could manage without it for now. On
• 8 further inquiry, she admits that she has been using it on a daily basis in the past month, which is why she ran out of it early. Her
• 9 friends arrive in the emergency department and seem like responsible, stable people. Which of the following is the most appropriate
• 10 next step in management?
• 11
• 12
• 13 (I 0 A. Admrt her to the medical unit and treat for delirium
• 14
0 B. Admit her to the psychiatry unit and treat the anxiety attack
• 15
• 16 0 C. Discharge her to her friends and advise her to return to the hospital if her condition worsens
• 17 0 D. Prescribe alprazolam for a week while she is in town and advise her to see her physician when she returns home
• 18
19
0 E. Suggest she endure the symptoms by going "cold turkey" off alprazolam

• 20
• 21
• 22 Explanation Rel<aps & Refs @ Help Q My Questions

• 23
The correct answer is A . This patient needs to be admitted because of the development of 45% of students got this correct.
• 24
delirium secondary to withdrawal from alprazolam. Delirium is treated as a medical emergency
• 25 Send us your feedback a t:
and needs to be monitored on a medical unit. Detoxification can be done there.
• 26 medFeedback@kaplan.com

• 27 Admitting to the psychiatry unit (choice B) to treat anxiety attacks is not justified . Please includE - -- - • --
• 28
• 29
• 30 Exit
• 1
• 2
• 3
0 A. Admit her to the medical unit and treat for delirium
• 4
• 5 0 B. Admit her to the psychiatry unit and treat the anxiety attack
• 6 0 c. Discharge her to her friends and advise her to return to the hospital if her cond ition worsens
• 7
0 D. Prescribe alprazolam for a week while she is in town and advise her to see her physician when she returns home
• 8
• 9 0 E. Suggest she endure the symptoms by going "cold turkey" off alprazolam
• 10
• 11
• 12 Explanation ReKaps & Refs @ Help Q My Questions
• 13
• 14 The correct answer is A . This patient needs to be admitted because of the development of 45% of students got this correct.
• 15 delirium secondary to withdrawal from alprazolam. Delirium is treated as a medical emergency
Send us your feedback at:
• 16 and needs to be monitored on a medical unit. Detoxification can be done there . medF eedback@kaplan. com
• 17
Admitting to the psychiatry unit (choice B) to treat anxiety attacks is not justified. Please include QID s2s157m
• 18
• 19 Discharging the patient to friends with the possibility of returning if she worsens (choice C) is
• 20 inappropriate care. The patient is already in withdrawal and is in danger of developing seizures.

• 21
Prescribing alprazolam (choice D) for the patient may not be enough to keep her out of
• 22
withdrawal because she has already been taking more than prescribed.
• 23
• 24 Going "cold turkey" (choice E) is not safe in a case of benzodiazepine withdrawal and can
• 25 end lethally.

• 26
• 27
• 28
• 29
• 30
• 1
• 2
• 3
• 4 A 45-year-old man who has paranoid schizophrenia is brought to the clinic by his mother. She reports that he has been followed at the
• 5 clinic for many years, treated with a number of different antipsychotic agents, and is currently on risperidone, 4 mg/d, and fluoxetine,
• 6 20 mg/d. His psychotic symptoms are well managed on risperidone. She is concerned about some abnormal movements that he has
• 7 developed in the past few months. These include facial grimacing and rolling movements of his fingers and toes. When she asks her
• 8 son about them, he says that he cannot stop them. Which of the following is the correct diagnosis for this condition?

• 9
• 10
0 A. Akathisia
• 11
• 12 0 B. Dystonia
• 13
0 C. Fluoxetine side effect
• 14
• 15 0 D. Par1<insonism
(I 0 E. TardiVe dyskinesia
• 17
• 18
• 19 Explanation Rel<aps & Refs 6 Help Q My Questions
• 20
• 21 The correct answer is E. This patient who has schizophrenia presents with facial grimacing, 84% of students got this correct
• 22 teeth grinding, and involuntary movements of his fingers and toes, which are all consistent with
• 23 Send us your feedback at:
tardive dyskinesia. Extrapyramidal symptoms are among the more serious side effects of medFeedback@kaplan.com
• 24 antipsychotic medication. This patient has likely developed tardive dyskinesia as a result of
25 Please include QIO m001058
• chronic treatment with antipsychotic medication .
• 26
Akathisia (choice A) is defined as a subjective feeling of restlessness that can occur
• 27 c:o,..nnr1ont tn ontinc:\/l'hntil" nr nth or ncut"hntrnnit" finl"'ll uiinn fl••nvotino \ ••co It I"Qn he trc~toA

• 28
• 29
• 30 Exit
• 1
• 2
• 3
0 E. Tard1ve dyskmesia
• 4
• 5
• 6
• 7 Explanation ReKaps & Refs t} Help Q My Questions

• 8
9 The c orrect answer is E. This patient who has schizophrenia presents with facial grimacing, 84% of students got this correct.

teeth grinding, and involuntary movements of his fingers and toes, which are all consistent with
• 10 Send us your feedback at:
tardive dyskinesia. Extrapyramidal symptoms are among the more serious side effects of
• 11 medFeedback@kaplan.com
antipsychotic medication. This patient has likely developed tardive dyskinesia as a result of
• 12 Please include QID m001058
chronic treatment with antipsychotic medication.
• 13
• 14 Akathisia (choice A) is defined as a subjective feeling of restlessness that can occur
• 15 secondary to antipsychotic or other psychotropic (including fluoxetine) use. It can be treated
with beta-blockers or anticholinergic agents.
• 17 Dystonia (choice B) can occur secondary to antipsychotic medication and usually occurs
• 18 within a few hours after taking the antipsychotic dose. Symptoms of dystonia include stiffness
• 19 and muscle spasms. The treatment includes antihistamines or anticholinergics .
• 20
Fluoxetine (choice C) can cause motor side effects, including akathisia and tremors, but does
• 21
not usually lead to tardive dyskinesia
• 22
• 23 Parkinsonism (choice D) can be a side effect of chronic antipsychotic medication use. It
• 24 presents with cogwheel rigidity, masked facies, bradykinesia, pill-rolling tremor, and a shuffling
• 25 gait. It is often treated with anticholinergic medication.
• 26
• 27
• 28
• 29
• 30
• 1
• 2
• 3
• 4 A 42-year-old man comes to the physician complaining of a 2-month history of a depressed mood, feelings of guilt and hopelessness,
• 5 difficulty sleeping, and a decreased appetite. When discussing medical therapy, the patient tells his physician that he is extremely
• 6 concerned about his sexual performance, as he is worried that he is getting older and that he is having some marital difficulties with
• 7 his wife. Wh ich of the following medications would be the best choice for initial treatment of this patient's condition?
• 8
• 9
• 10
(j 0 A. Buprop1on

• 11 0 B. Fluoxetine
• 12
0 C. Imipramine
• 13
• 14 0 0 . Paroxetine
• 15 0 E. Sertraline
• 16
• 2.7
• 18
Explanation ReKaps & Refs @ Help 0 My Questions
• 19
• 20 The correct answer is A. The diagnosis is major depression, because of symptoms such as: 28% of students got this correct.
• 21
• 22 • Depressed mood Send us your feedback at:
• Feelings of guilt and hopelessness medFeedback@kaplan.com
• 23
• Difficulty sleeping Please include QJO Q0396m
• 24
• Decreased appetite
• 25
• 26 Of the answer choices listed, bupropion is the antidepressant that has the fewest adverse sexual
• 27 side effects.
• 28
• 29
• 30 Exit
• 1
• 2
• 3
• 4
0 D. Paroxetine
• 5
• 6 0 E. Sertraline
• 7
• 8
• 9 Explanation ReKaps & Refs @ Help 0 My Questions
• 10
• 11 The correct answer is A. The diagnosis is major depression, because of symptoms such as: 28% of students got this correct.

• 12
• Depressed mood Send us your feedback at:
• 13 • Feelings of guilt and hopelessness medFeedback@kaplan.com
• 14 • Difficulty sleeping Please include QID Q0396m
• 15 • Decreased appetite
• 16
Of the answer choices listed, bupropion is the antidepressant that has the fewest adverse sexual
• 2.7
side effects.
• 18
• 19 Selective serotonin reuptake inhibitors such as fluoxetine (choice B) . paroxetine (choice D) .
• 20 and sertraline (choice E) are all known to be associated with erectile and orgasmic
• 21 disturbances such as delayed ejaculation and anorgasmia.
• 22 Because imipramine (choice C) also has an effect on the serotonin levels in the brain, it too
• 23 has sexual side effects, although to a somewhat lesser degree than the SSRis.
• 24
• 25 Reviewed on 1216112
• 26
• 27
• 28
• 29
• 30
• 1
• 2
• 3
4 A 35-year-old woman comes to her primary care physician for her routine yearly heaHh maintenance examination. She denies any

new problems. Her only significant medical history includes a 5-year history of schizophrenia, which has been well-controlled with
• 5
antipsychotic agents; and a 5-year history of hypertension, for wh ich she takes a diuretic. Vital signs and physical examination are
• 6
within normal limits. However, the patient is noted to have grimacing and rapid eye blinking. She denies having noticed the abnormal
• 7
movements, and her speech is normal. Which of the following accurately describes this condition?
• 8
• 9
• 10 0 A. Adding benztropine will treat the condition
• 11
0 B. Start treatment with bromocriptine
• 12
• 13 0 C. Stopping the medication immediately will treat the problem
• 14
0 D. Using benzodiazepines will treat the condition
• 15
• 16 (I 0 E. Ustng newer antipsychotics would have been less likely to have caused thiS condition
• 17

• 19 Explanation Rel<aps & Refs f) Help 0 My Questions


• 20
• 21 The correct answer is E. Tardive dyskinesia (TO) is a syndrome characterized by abnormal 73% of students got th is correct.

• 22 choreiform and athetoid movements occurring late in onset in relation to initiation of


Send us your feedback at:
• 23 antipsychotic treatment. TO usually develops after months to years of treatment with medFeedback@kaplan.com
• 24 antipsychotic agents, and is presumably much less likely to occur with the use of atypical
Please include QIO Q0371m
• 25 antipsychotics. Risk factors form include older age, longer duration of antipsychotic treatment,

26 and presence of an affective disorder. The abnormal involuntary movements usually involve

orofacial muscles but may include the trunk and extremities. Regular examinations should be
• 27 ... ........... . ................... .... .... ....... : ........... ·---·-""' ••• :.&.. 1&-. .................... ...a: .........: ............................ ............... _; ___ ... c Tr\ ................ ....

• 28
• 29
• 30 Exit
• 1
• 2
• 3
• 4
• 5 The corr ect answer is E. Tardive dyskinesia (TD) is a syndrome characterized by abnormal 73% of students got this correct

• 6 choreiform and athetoid movements occurring late in onset in relation to initiation of


Send us your feedback at:
• 7 antipsychotic treatment. m usually develops after months to years of treatment with medFeedback@kaplan.com
• 8 antipsychotic agents, and is presumably much less likely to occur with the use of atypical
Please include QID Q037lm
antipsychotics. Risk factors form include older age, longer duration of antipsychotic treatment,
• 9
and presence of an affective disorder. The abnormal involuntary movements usually involve
• 10
orofacial muscles but may include the trunk and extremities. Regular examinations should be
• 11
done to ensure that patients treated with these medications do not develop signs of m because
• 12
once developed, the condition is irreversible.
• 13
• 14 Since the condition is irreversible, all of the answer choices that mention a way to treat the
• 15 condition (choices A, B, and D) are wrong.

• 16 Benztropine (choice A) will treat acute dystonia, which is another side effect of
• 17 antipsychotics .

Bromocriptine (choice B ) is a dopmamine agonist that can be used in the treatment of


neuroleptic malignant syndrome often associated with high potency typical antipsychotics. It is
prescribed to restore the dopamine tone.

• 22 Stopping the medication (choice C) will not treat the TO, but should be done anyway.
• 23
Benzodiazepine (choice D) will treat akathisia, which is another side effect of antipsychotics.
• 24
• 25 Reviewed on 12/10112
• 26
• 27
• 28
• 29
• 30
• 1
• 2
• 3
• 4 A 36-year-old man with a history of hypertension, hypercholesterolemia , and bipolar disorder presents with a 2-week history of facial
• 5 twitching and tremors of his hands. He has also had nausea and diarrhea for the last few days. He has had to take a leave from work

6 due to his symptoms. He was diagnosed as having hypertension and hypercholesterolemia 6 years ago. His bipolar disorder was

7 diagnosed 1 year ago and his symptoms have responded well to therapy. Physical examination shows a temperature of 36.2"C

(97.1°F), blood pressure of 130/70 mm Hg, respiratory rate of 14/min, and heart rate of 50/min. Pupils are equal, round , and reactive
• 8
to light. Nystagmus is present. Fundus examination shows no papilledema. There is no neck rigidity or lymphadenopathy. Lungs are
• 9
clear to auscultation, with no wheezes or crackles. Cardiovascular examination shows normal S1 and S2 sounds with no rub or gallop.
• 10
Abdomen is soft, nontender, and nondistended. Bowel sounds are active. Irregular coarse tremors are seen in both upper extremities.
• 11
Motor strength is normal. There are no sensory deficits. Lab results are:
• 12
• 13 Hgb 14.2 g/dl
• 14 WBC 12,700/mm 3
• 15 Platelets 320,000/mm 3
• 16
Sodium 138 mEq/l
• 17 Potassium 4.4 mEq/l
• 18 Chloride 102 mEq/l
• 19 Bicarbonate 26 mEq/l
• 20 BUN 21 mg/dl
• 21 Creatinine 0.9 mg/dl
• 22 Glucose 80 mg/dl
• 23 Calcium 9.6 mg/dl

• 24 CT scan of the head is normal. Which of the following is the most appropriate diagnostic test?
• 25
• 26
• 27 l oA. Fasting lipid profile
• 28
• 29
• 30 Exit
• 1
• 2
• 3
• 4 CT scan of the head is normal. Which of the following is the most appropriate diagnostic test?
• 5
• 6
0 A. Fasting lipid profile
• 7
• 8 0 B. Glycosylated hemoglobin
• 9
f) 0 C. Lllh1um level
• 10
• 11 0 D. Nerve conduction studies
• 12 0 E. No further tests are needed
• 13
14 0 F. Repeat CT scan of the head

• 15
• 16
• 17 Explanation ReKaps & Refs 0 Help 0 My Questions

• 18
The correct answer is C. Lithium has a low therapeutic index. Findings of lithium toxicity that 90% of students got this correct
• 19
may be observed include neuromuscular excitability, irregular coarse tremors, motor agitation,
• 20 Send us your feedback at:
21 muscle weakness, ataxia, sluggishness, delirium, nausea, vomiting, diarrhea, leukocytosis, sinus
• medFeedback@kaplan. com
bradycardia, and hypotension. Severe lithium intoxication can lead to seizures, stupor, and
• 22 Please include QIO m000574m
coma. II could also lead to permanent neurologic sequelae. There is good correlation between
• 23
symptoms and blood levels. Given this patient's clinical features (facial twitching, coarse
• 24
tremors, nausea, diarrhea, sinus bradycardia, and leukocytosis), it would be appropriate to
• 25
obtain a serum lithium level.
• 26
• 27 The patient does have history of hyperlipidemia. However, a lipid profile (choice A) at this time
• 28
• 29
• 30 Exit
• 1
• 2
• 3
• 4 Explanation ReKaps & Refs ti Help 0 My Questions
• 5
• 6 The correct answer is C. Lithium has a low therapeutic index. Findings of lithium toxicity that 90% of students got this correct.
• 7 may be observed include neuromuscular excitability, irregular coarse tremors, motor agitation,
• 8 Send us your feedback at:
muscle weakness, ataxia, sluggishness, delirium, nausea, vomiting, diarrhea, leukocytosis, sinus medFeedback@kaplan.com
• 9 bradycardia, and hypotension. Severe lithium intoxication can lead to seizures, stupor, and
• 10 Please include QID m000574m
coma. II could also lead to permanent neurologic sequelae. There is good correlation between
• 11 symptoms and blood levels. Given this patienfs clinical features (facial twitching, coarse
• 12 tremors, nausea, diarrhea, sinus bradycardia, and leukocytosis), it would be appropriate to
• 13 obtain a serum lithium leveL
• 14 The patient does have history of hyper1ipidemia. However, a lipid profile (choice A) at this time
• 15 will not be helpful in determining the etiology of his symptoms.
• 16
• 17 The patient is not a diabetic. Checking a glycosylated hemoglobin level (choice B ) would not

• 18 be helpful.

• 19 Nerve conduction studies (choice D) are helpful in detection of neuropathies but would not be
• 20 helpful in detecting lithium toxicity .
• 21
If left untreated, lithium toxicity can be fatal Not ordering further tests, including a lithium level
• 22
(choice E) to detect the presence of lithium toxic ity, would not be appropriate.
• 23
• 24 The patient has had a normal CT scan of the head. Repeating the CT scan (choice F) is not
• 25 likely to be helpful.

• 26
• 27
• 28
• 29
• 30
• 1
• 2
• 3
4 A young woman comes to a physician for help. During the interview, she reveals that she was raped when she was a teenager. She is

currently dating a man with whom she would like to have sexual intercourse. When they tried, however, she felt "uptight" and they
• 5
could not have intercourse. Which of the following is the most likely treatment of this condition?
• 6
• 7
• 8 0 A. Couples therapy
• 9
• 10 (I 0 B. Kegel exercises

• 11 0 C. Squeeze and stop-andiJO


• 12
O D.SSRI
• 13
• 14
• 15
• 16 Explanation ReKaps & Refs 0 Help Q My Questions

• 17
The corr ect answer is B . Vaginismus is one of the sexual pain disorders. It is defined as 47% of students got this correct.
• 18
involuntary muscle contraction of the outer third of the vagina that interferes with intercourse. It
• 19 Send us your feedback at:
is prevalent in women who have a history of sexual trauma, emotional abuse, rigid religious medFeedback@kaplan. com
upbringing, or psychosexual conflicts. It is treated with the help of Kegel exercises and dilators.
Please include QID Q0786m
• 22 Couples therapy (choice A) is indicated only when there is a marital conflict.
• 23
Squeeze and stop-andiJO (choice C) is the treatment for premature ejaculation in males,
• 24 which is characterized by ejaculation before the man wishes to do so.
• 25
• 26 An SSRI (c hoice D) should not be the first choice in therapy. It may be used when behavioral
• 27 therapy fails.

• 28
• 29
• 30 Exit
• 1
• 2
• 3
A woman comes to the clinic with her 13-year-old grandson, stating that he has had behavioral problems for the past 4 years. She
• 4
says that she has been trying to handle this issue on her own, but it is getting out of control. Her grandson is living with her now
• 5
because his mother is in a drug rehabilitation center. She describes that the boy back-answers in class and argues with teachers,
• 6 adult neighbors, and herself. He often blurts out answers to questions before the questions are completed. He is hyperactive and
• 7 impulsive. He deliberately argues to annoy people. He has been kicked off the school bus for lying about another student and
• 8 challenging the driver. Recently, she caught him stealing from her wallet. He is about to be expelled from school, since he hit his
• 9 classmate. Which of the following is the most likely diagnosis?
• 10
• 11
• 12 O A.ADHD
• 13 0 B. Antisocial personality disorder
• 14
• 15 0 0 C. Conduct disorder
• 16 0 D. Intermittent explosive disorder
• 17
0 E. Oppositional defiant disorder
• 18
• 19
• 20
• 21.
Explanation ReKaps & Refs 8 Help Q My Questions

• 22
The correct answer is C. Conduct disorder is a childhood/adolescent disorder in which the 73% of students got this correct.
• 23
basic rights of others are violated, with a few of the following present in the past 12 months:
• 24 Send us your feedback a t:
• 25 • Destruction of property medFeedback@kaplan.com

• 26 • Cruelty to animals and people (this patient hit his classmate) Please include QIO Q0080m
• 27 • Deceitfulness or theft (he lied and was caught stealing)
• 28
• 29
• 30 Exit
• 1
• 2
• 3
• 4 The correct answer is C. Conduct disorder is a ch ildhood/adolescent disorder in which the 73% of students got this correct.

• 5 basic rights of others are violated, with a few of the following present in the past 12 months:
Send us your feedback at:
• 6 medFeedback@kaplan.com
• Destruction of property
• 7
• Cruelty to animals and people (this patient hit his classmate) Please include QID Q0080m
• 8 • Deceitfulness or theft (he lied and was caught stealing)
• 9 • Serious violations of rules
• 10
Conduct disorder causes clinically significant impairment in social functioning, and it is
• 11
reserved for patients younger than 18 years. Patients with conduct disorder may have some
• 12
overlapping features of oppositional defiant disorder and ADHD, as in this patient.
• 13
• 14 ADHD (choice A) requires the presence of:
• 15
• six symptoms of inattention for at least 6 months to a degree that is maladaptive, and
• 16
• six symptoms of hyperactivity/impulsivity that cause social impairment.
• 17
• 18 Symptoms are present in two or more settings (e g • home and school), and some of the
19 symptoms are present before age 7. This patient only blurts out answers to questions before the

questions are completed and is a little hyperactive and impulsive. He does not meet the criteria
• 20
for the diagnosis of ADHD.
• 21
• 22 If the patient were to present with the same symptoms after the age of 18, antisocial personality
• 23 disorder (choice B) would be the diagnosis.
• 24
Intermittent explosive disorder {choice D) is diagnosed in adults only after several episodes of
• 25
failure to resist aggressive impulses that lead to assaults or destruction of property. The degree
• 26 of episodes is not proportionate to the precipitating stressor. This patient's aggression is
• 27 neither intermittent nor explosive.
• 28
• 29
• 30 Exit
• 1
• 2
• 3
• 4
ADHD (choice A) requires the presence of:
• 5
• 6 • six symptoms of inattention for at least 6 months to a degree that is maladaptive, and
• 7 • six symptoms of hyperactivity/ impulsivity that cause social impairment.
• 8 Symptoms are present in two or more settings (e.g., home and school), and some of the
• 9 symptoms are present before age 7. This patient only blurts out answers to questions before the
• 10 questions are completed and is a little hyperactive and impulsive. He does not meet the c riteria
• 11 for the diagnosis of ADHD.
• 12
If the patient were to present with the same symptoms after the age of 18, antisocial personalily
• 13
disorder (choice B) would be the diagnosis.
• 14
• 15 Intermittent explosive disorder (choice D) is diagnosed in adults only after several episodes of
• 16 failure to resist aggressive impulses that lead to assaults or destruction of property. The degree
• 17 of episodes is not proportionate to the precipitating stressor. This patient's aggression is
• 18 neither intermittent nor explosive.
• 19 Oppositional defiant disorder (ODD; choice E ) is a pattern of negativistic and defiant
• 20 behavior lasting at least 6 months with four or more of the following: loss of temper, arguments
• 21 with adults, defying rules, deliberately annoying other people, blaming others for own faults,
• 22 presence of vindictive behavior, presence of anger, and resentment. But patients with ODD do
• 23 not violate social norms. They only argue and annoy people, especially authoritative paras. The
• 24 patient in this question has gone beyond just annoying people: he hit people, he lied, he was
• 25 caught stealing. So the diagnosis is conduct disorder.
• 26
• 27
• 28
• 29
• 30
• 1
• 2
• 3
A 22-year-old woman with anorexia nervosa and a history of depression, for which she is currently taking bupropion, moves to a new
• 4
city and goes to a local psychiatrist to establish care in her new location. She tells the doctor that she has been on the same dose of
• 5 bupropion since she was 18 years old, and that her depression has been stable, but that she has lost 10 pounds over the last month
• 6 and weighs only 90 pounds (41 kg) now with a height of 5 feet 5 inches (165 em). The physician is concerned and explains that there
• 7 is a complication that she must be aware of if she is going to continue with this medication. About which of the following complications
• 8 is the physician most likely concerned?
• 9
• 10
• 11 0 A. Cardiac arrhythmia
• 12
0 B. Decreased renal function
• 13
• 14 0 C. Hypochondriasis
• 15 0 D. Osteoporosis
• 16
• 17 (I 0 E. Se1zures

• 18
• 19
• 20 Explanation Rel<aps & Refs G Help Q My Questions

• 21
The correct answer is E. Bupropion is an antidepressant that can lower the seizure threshold 39% of students got this correct.
for some patients, and as a result should be discontinued or carefully monitored in a patient with
• 23 Send us your feedback at:
anorexia nervosa. Patients with anorexia are at increased seizure risk related to large nuid shifts
• 24 medFeedback@kaplan.com
and electrolyte disturbances caused by bingeing and purging behavior.
• 25 Please include QIO Q0275
• 26 Wh ile cardiac arrhythmia (c hoice A) is an important medical complication of anorexia
• 27 nervosa, bupropion has not been demonstrated to adversely affect cardiac function .
• 28
• 29
• 30 Exit
• 1
• 2
• 3
• 4
• 5 (I 0 E. Se1zures
• 6
• 7
• 8 Explanation ReKaps & Refs @ Help Q My Questions
• 9
• 10 The c orrect answer i s E. Bupropion is an antidepressant that can lower the seizure threshold 39% of students got this correct.

• 11 for some patients, and as a result should be discontinued or carefully monitored in a patient with
Send us your feedback at:
• 12 anorexia nervosa. Patients with anorexia are at increased seizure risk related to large fluid shifts
medFeedback@kaplan.com
• 13 and electrolyte disturbances caused by bingeing and purging behavior.
Please include QID Q0275
• 14 While cardiac arrhythmia (choice A) is an important medical complication of anorexia
• 15 nervosa, bupropion has not been demonstrated to adversely affect cardiac function .
• 16
Decreased renal function (choice B) is not typically a medical complication of anorexia
• 17
nervosa, and is also not attributable to the use of bupropion
• 18
• 19 Hypochondriasis (choice C) is a disorder that involves preoccupation with fears of having a
• 20 serious disease based on the patient's misinterpretation of bodily symptoms, and is not related
• 21 to the use of bupropion in depressed patients

Osteoporosis (choice D) is a chronic long term effect of anorexia nervosa, but is not related to
• 23 the use of bupropion.
• 24
• 25 Reviewed on 12/10112
• 26
• 27
• 28
• 29
• 30
• 1
• 2
• 3
• 4 A 19-year-old student is admitted to the psychiatry inpatient unit with the working diagnosis of psychosis. Because of combativeness

• 5 and threats to others, he is put in seclusion. At first, he refuses to take medication by mouth; however, after attacking a nurse he is
given medication intramuscularly on two occasions. He has now developed a painful contraction of a group of muscles in the neck and
• 6
twitching of the mouth and face. Which of the following is the most appropriate treatment?
• 7
• 8
• 9
0 A. Clozapine
• 10
• 11 0 8 . Dantrolene
• 12 o c. Diazepam
• 13
• 14 (I O D. Diphenhydramine

• 15 0 E. Hydralazine
• 16
• 17
• 18 Explanation ReKaps & Refs 0 Help 0 My Questions
• 19
• 20 The correct answer is D. Acute dystonia or dystonic reaction is an involuntary spasm of a 34% of students got this correct.
• 21 particular group of muscles that can involve the neck, jaw, tongue, eyes, or entire body. It can
Send us your feedback at:
• 22 be an early adverse effect of antipsychotics, and is more common in younger men. It is more
medFeedback@kaplan.com
common with typical antipsychotics. The treatment of choice is parenteral administration of
Please include QJO Q0464m
• 24 anticholinergics such as diphenhydramine, benztropine, and trihexyphenidyl.
• 25 Clozapine (choice A) is a newer antipsychotic and less likely to cause dystonic reactions.
• 26 These side effects are more common in the typical antipsychotics such as haloperidol
• 27
• 28
• 29
• 30
• 1
• 2
• 3
• 4 0 E. Hydralazine
• 5
• 6
• 7 Explanati on ReKaps & Refs @ Help 0 My Questions
• 8
• 9 The c orrect answer i s D. Acute dystonia or dystonic reaction is an involuntary spasm of a 34% of students got this correct
• 10 particular group of muscles that can involve the neck, jaw, tongue, eyes, or entire body. It can
Send us your feedback at:
• 11 be an early adverse effect of antipsychotics, and is more common in younger men. It is more medFeedback@kaplan.com
• 12 common with typical antipsychotics. The treatment of choice is parenteral administration of
Please include QID Q0464m
• 13 anticholinergics such as diphenhydramine, benztropine, and trihexyphenidyl.
• 14 Clozapine (choice A) is a newer antipsychotic and less likely to cause dystonic reactions .
• 15 These side effects are more common in the typical antipsychotics such as haloperidol
• 16
Dantrolene (choice B ) is used in the treatment of neuroleptic malignant syndrome, which is a
• 17
life-threatening adverse effect of antipsychotics that is characterized by muscle rigidity.
• 18
hyperthermia, autonomic instability, and delirium.
• 19
• 20 Diazepam (choice C) may be used in the treatment of akathisia, which is another adverse
• 21 effect of antipsychotics, characterized by motor restlessness.
• 22
Hydralazine (choice E) is a direct-acting smooth muscle relaxant used to treat hypertension
by acting as a vasodilator primarily in arteries and arterioles
• 24
• 25 Reviewed on 12/10112
• 26
• 27
• 28
• 29
• 30
• 1
• 2
• 3
Three hours after an uneventful diverticulectomy, a 78-year-old HIV-positive man becomes disoriented and confused. He repeatedly
• 4
asks the nurses where he is, and his speech pattern is disorganized and rambling. His temperature is 104°F, blood pressure is 120/80
• 5
mm Hg, pulse is 88/min, and respirations are 18/min. The patient is uncooperative, but shows no physical abnormalities. He has a
• 6 family history of schizophrenia. He smokes half a pack of cigarettes per day and had a history of alcohol abuse 10 years ago. Mental
• 7 status examination is not possible because of an aHered level of consciousness. Laboratory studies show:
• 8
• 9 RBC: 5.2 million cells/mel
WBC: 10,500
• 10
Hematocrit 43%
• 11
Hemoglobin: 14gm/dl
• 12
• 13 ABG shows:
• 14 pH: 7.42
• 15 PaCO 2 : 39 mm Hg
• 16 PaO 2 : 94 mm Hg
• 17
HCO 3 : 24 mEq/L
• 18
• 19 Which of the following is the most likely diagnosis?
• 20
• 21
• 22 0 A. Brief psychotic disorder
• 23 0 B. Delirium tremens
0 C. Dementia
• 25
• 26 f) 0 D. Febrile delirium
• 27 () 1= Hvnmi~ r1Aiirium
• 28
• 29
• 30 Exit
• 1
• 2
• 3
• 4 0 C. Dementia
• 5
• 6 fl 0 D. Febnle delirium

• 7 0 E. Hypoxic delirium
• 8
• 9
• 10 Explanation ReKaps & Refs 6 Help Q My Questions
• 11
• 12 The correct answer is D. This patient has delirium, which is also called acute confusional 20% of students got this correct.
• 13 state. It is very common in hospitalized and institutionalized elderly indMduals. Moreover, he is
Send us your feedback at:
• 14 febrile , so fever should be the cause of this delirium. Delirium is characterized by a rapid onset medFeedback@kaplan.com
• 15 of impaired cognition, a~ered level of consciousness, disturbances in attention and psychomotor
Please include QID Q0753m
• 16 actMty, and a~ered sleep-wake cycles. The symptoms tend to fluctuate and are usually
• 17 reversible when the underlying disorder is identified and treated. Other common causes include
• 18 psychological and physical stress (for example, surgery), metabolic disturbances, neoplasms,
• 19 infections, medications, cerebral and cardiovascular diseases, and withdrawal from alcohol
• 20 and/or prescription medications.

• 21 Brief psychotic disorder (choice A) is characterized by the abrupt onset of psychotic


symptoms, including hallucinations, delusions, and disorganization with impaired functioning . It
is present for more than 1 day but less than 1 month. It is usually preceded by a stressful life
event. The family history of schizophrenia that is given in the stem is used as a distracter.
• 25
Delirium tremens (choice B) is something to consider, but this patient has a history of alcohol
• 26
abuse, not dependence, in the distant past. Therefore, this diagnosis is less likely.
• 27
• 28
• 29
• 30
• 1
• 2
• 3 e aps es
• 4
• 5 The correct answer is D. This patient has delirium, which is also called acute confusional 20% of students got this correct
• 6 state. It is very common in hospitalized and institutionalized elderly individuals. Moreover, he is
Send us your feedback at:
• 7 febrile, so fever should be the cause ofthis delirium. Delirium is characterized by a rapid onset medFeedback@kaplan.com
• 8 of impaired cognition, altered level of consciousness, disturbances in attention and psychomotor
Please include QIO Q0753m
• 9 activity, and altered sleep-wake cycles. The symptoms tend to fluctuate and are usually
• 10 reversible when the underlying disorder is identified and treated. other common causes include
• 11 psychological and physical stress (for example, surgery}, metabolic disturbances, neoplasms,
• 12 infections, medications, cerebral and cardiovascular diseases, and withdrawal from alcohol
• 13 and/or prescription medications.
• 14 Brief psychotic disorder (choice A) is characterized by the abrupt onset of psychotic
• 15 symptoms, including hallucinations, delusions, and disorganization with impaired functioning . It
• 16 is present for more than 1 day but less than 1 month. It is usually preceded by a stressful life
• 17 event. The family history of schizophrenia that is given in the stem is used as a distracter.
• 18
Delirium tremens (choice B) is something to consider, but this patient has a history of alcohol
• 19
abuse, not dependence, in the distant past. Therefore, this diagnosis is less likely .
• 20
• 21 Dementia (choice C) refers to the slow and insidious onset of cognitive and intellectual deficits
with no changes in consciousness. The symptoms are stable and irreversible. The most
common causes are Alzheimer disease and multi-infa rct dementia.

Hypoxic delirium (choice E) is another possibility, but in this case has been excluded by a
• 25 normal ABG .
• 26
• 27
• 28
• 29
• 30
• 1
• 2
• 3
A 28-year-old man comes to the emergency department complaining of a 3-month history of fatigue, depression, insomnia,
• 4
guilt-feeling , and weight loss. A Mini-Mental Status examination reveals that he has some mild cognitive defects, scoring 24 of 30. On
• 5
physical examination he is noted to have oral thrush and lymphadenopathy. Which of the following is the most appropriate next step in
• 6
management?
• 7
• 8
• 9 0 A. Administer fluoxetine
• 10
0 B. Administer imipramine
• 11
• 12 fl 0 C. Order an ELISA
• 13
0 D. Order a urine drug screen
• 14
• 15 0 E. Head CT without contrast
• 16
• 17
• 18 Explanation ReKaps & Refs G Help Q My Questions
• 19
• 20 The correct answer is C. Given this patient's findings of lymphadenopathy and oral thrush, 54% of students got this correct.

• 21 A IDS is the most likely diagnosis in the context of mild cognitive deficits and mood disturbance.
Send us your feedback at:
• 22 Thus, an HIV ELISA should be ordered first to make the diagnosis of A IDS. medFeedback@kaplan.com
• 23 Before beginning treatment with antidepressant medication, mood disturbance due to a general Please include QJO Q0406m
• 24 medical condition or organic causes must be ruled out. Thus, ordering fluoxetine (c hoice A) or
• 25 imipramine (c hoice B) is inappropriate at this time .
• 26
Urine drug screen (choice D) may prove helpful to find out if IV drug abuse is the cause of the
• 27 /\InC: ..u.t ir ,...,.,...,.,. ,..,,,,,... hnr"th~n ~n 1-11\/tn.rt
• 28
• 29
• 30 Exit
• 1
• 2
• 3
• 4 0 B. Administer imipramine
• 5
(I 0 C. Order an ELISA
• 6
• 7 0 D. Order a urine drug screen
• 8 0 E. Head CT without contrast
• 9
• 10
• 11 Explanation ReKaps & Refs @ Help Q My Questions
• 12
• 13 The correct answer is C. Given this patienrs findings of lymphadenopathy and oral thrush, 54% of students got tflis correct.
• 14 AIDS is the most likely diagnosis in the context of mild cognitive deficits and mood disturbance.
• 15 Thus, an HIV ELISA should be ordered first to make the diagnosis of AIDS.
Send us y our feedback at:
medF eedback@kaplan. com
• 16
17 Before beginning treatment with antidepressant medication, mood disturbance due to a general Please include QJO Q0406m

medical condition or organic causes must be ruled out Thus, ordering fluoxetine (choice A} or
• 18
imipramine (choice B) is inappropriate at this time.
• 19
• 20 Urine drug screen (choice D) may prove helpful to find out if IV drug abuse is the cause of the
• 21 A IDS but is of less value here than an HIV test
• 22 CT without contrast may be indicated after establishing the diagnosis of HIV and with complaints
• 23 of headache or focal neurologic deficit (choice E) .
• 24
• 25 Reviewed on 12/10112
• 26
• 27
• 28
• 29
• 30
• 1
• 2
• 3
• 4 A 49-year-old veteran comes to the physician to discuss smoking cessation. He was recently diagnosed with high blood pressure,
• 5 hypercholesterolemia, and coronary artery disease, and realizes that it is finally time to quit. He has been a heavy smoker for years

6 and finds it impossible to stop on his own, but his wife has heard about the use of bupropion for the treatment of nicotine dependence.

7 After discussing the pros and cons of starting bupropion, the patient agrees to give it a try. Which of the following instructions should

be given to the patient about setting a date to stop smoking?
• 8
• 9
• 10 (I 0 A. He should decrease smoking gradually during the second week of treatment wrth bupropron
• 11
• 12 0 B. He should set a date after reaching the therapeutic dose of 450 mg
• 13 0 C. He should stop smoking on the third day after starting bupropion
• 14
0 0 . He should stop smoking right away
• 15
• 16 0 E. His stopping date should be after 6 months of treatment with bupropion
• 17
• 18
• 19 Explanation Rel<aps & Refs G Help Q My Questions
• 20
• 21 The correct answer is A. Instructions to the patient regarding smoking cessation once 82% of students got this correct .
• 22 medication treatment has started include gradual discontinuation of smoking during the second
Send us your feedback at:
• 23 week of treatment while the dose is titrated up. medFeedback@kaplan .com
• 24
A dose of 450 mg (choice B) is usually high for smoking cessation treatment. The usual dose Please include QIO s2s145
• 25 is 300 mg and at that point smoking should be stopped. Higher doses carry a risk of side effects
and are more likely to be used in the treatment of severe depression.
• 27
• 28
• 29
• 30
• 1
• 2
• 3
• 4
0 D. He should stop smoking right away
• 5
• 6 0 E. His stopping date should be after 6 months of treatment with bupropion
• 7
• 8
• 9 Explanation ReKaps & Refs 0 Help 0 My Questions
• 10
• 11 The correct answer is A. Instructions to the patient regarding smoking cessation once 82% of students got this correct.

• 12 medication treatment has started include gradual discontinuation of smoking during the second
Send us your feedback at:
• 13 week of treatment while the dose is titrated up. medFeedback@kaplan.com
• 14 A dose of 450 mg (choice B) is usually high for smoking cessation treatment. The usual dose Please include QID s2s145
• 15 is 300 mg and at that point smoking should be stopped. Higher doses carry a risk of side effects
• 16 and are more likely to be used in the treatment of severe depression .
• 17
Stopping smoking on the third day (choice C) is not appropriate, because the medication has
• 18
not started to work yet.
• 19
• 20 Stopping smoking right away (choice D) is not appropriate, because the medication has not
• 21 started to work yet.
• 22 Stopping smoking after 6 months of treatment (choice E) with bupropion is not appropriate. It
• 23 should be done within the initial weeks of treatment.
• 24
• 25 Reviewed on 12/10112
' .
'
----""'------ '--- - -

• 27
• 28
• 29
• 30
• 1
• 2
• 3 was exE~cuuve as a was
• 4 shocked and angered at being let go. He remained calm and stoic when his boss gave him the news and thanked him for a wonderful
• 5 career. He arrives home that evening; his wife wants to know about his day and his children ask him for help with their homework. He
• 6 is tired and irritable. He criticizes his wife for making a poor dinner, throws his food away before eating it, and tells his children he
• 7 cannot help them tonight and they need to do their own work. Which of the following defense mechanisms is this man using?
• 8
• 9
10 0 A. Acting out

• 11 0 B. Conversion
• 12
• 13
0 0 C. Displacement

• 14 0 D. Dissociation
• 15
0 E. Rationalization
• 16
• 17
• 18
19
Explanation ReKaps & Refs ~ Help 0 My Questions

• 20
The correct answer is C. Defense mechanisms are unconscious attempts to decrease strong 86% of students got this correct.
• 21
emotions in the face of stressful circumstances. In this case, the husband is clearly upset about
• 22 being let go from his job. He is calm at the office but reacts to the situation by turning his anger
Send us your feedback at:
medFeedback@kaplan.com
• 23 and hostility on his wife and children at home. He is in effect "displacing" his anger and
• 24 frustration on his family instead of his employer. Please include QID m00 1049

• 25
Acting out (c hoice A) is in immature defense mechanism. It is the expression of an
• 26
unconscious impulse through a physical action. An example would be the man having a
• 71 ....,,.. ........ ........ .. ,.. , ..........
~ ~ ...................... ~,.. ........... '" ...... .......................
,.. ,..; ..... ,.. ........... . .
• 28
• 29
• 30 Exit
• 1
• 2
• 3
• 4
• 5 Explanation Rel<aps & Refs ~ Help Q My Questions

• 6
• 7 The correct answer is C. Defense mechanisms are unconscious attempts to decrease strong 86% of students got this correct.

• 8 emotions in the face of stressful circumstances. In this case, the husband is clearly upset about
Send us your feedback at:
9 being let go from his job. He is calm at the office but reacts to the situation by turning his anger
• medFeedback@kaplan.com
and hostility on his wife and children at home. He is in effect "displacing" his anger and
• 10 Please include QID m001049
frustration on his family instead of his employer.
• 11
• 12 Acting out (choice A) is in immature defense mechanism. It is the expression of an
• 13 unconscious impulse through a physical action. An example would be the man having a
• 14 behavioral outburst when he found out that he was being let go.
• 15 Conversion (choice B) is when emotional conflicts are transformed into physical symptoms.
• 16 An example would be the man having arm paraly sis or chest pain after finding out about his job.
• 17
Dissociation (choice D) is the blocking off of disturbing thoughts from consciousness in an
• 18
attempt to avoid negative or upsetting feelings An example would be if the husband blocked out
• 19
what had happened to him and came home and told his wife he had a wonderful day at the
• 20
office.
• 21
• 22 Rationalization (choice E) means giving a logical explanation for an upsetting circumstance
• 23 rather than the true reason . An example would be if the husband came home and told his wife
• 24 that he was downsized and that this was a good thing, because it would let him pursue other
• 25 business opportunities, such as writing a film.
• 26
• XI
• 28
• 29
• 30
• 1
• 2
• 3
• 4 An 18-year-old Caucasian woman is brought to the emergency department by her parents, who are concerned about her recent
• 5 weight loss. She has become a picky eater, avoiding foods that are high in calories, she exercises 2 hours per day , and she is
• 6 constantly saying that she fears food and "looks fat." Her height is 64 inches and her weight has dropped from 125 lb to 11 0 lb over
• 7 the past 4 months. She has not gotten her menstrual period in the past 3 months. She denies binge eating or purging symptoms. Vital
• 8 signs, physical examination, CBC, and electrolyte panel are within normal limits. What is the appropriate next step in the management

• 9 of this young woman?

• 10
• 11
0 A. Hospitalization
• 12
• 13 fl 0 B. Recommend a behaviorally-based outpatient treatment
• 14 0 C. Recommend long-term psychodynamic psychotherapy
• 15
• 16 0 D. Start clonazepam
• 17 0 E. Start fluoxetine
• 18
• 19
• 20 Explanation Rel<aps & Refs 0 Help 0 My Questions
• 21
• 22 The correct answer is B . This young girl presents with low weight, fear of becoming fat, and 28% of students got this correct
• 23 amenorrhea, which are consistent with a diagnosis of anorexia nervosa . She denies binge
• 24 Send us your feedback at:
eating or purging behaviors, so she meets the criteria for the restricting subtype of the illness.
medFeedback@kaplan.com
• 25 She has lost a significant amount of weight and has amenorrhea for 3 months. Her vital signs,
Please include QIO m001061
• 26 physical examination, and blood work are within normal limits. It is appropriate for her to start a
• 27 structured, behaviorally-based outpatient treatment to try to normalize her eating patterns and

• 29
• 30 Exit
• 1
• 2
• 3
• 4
• 5 Explanation ReKaps & Refs 6 Help 0 My Questions

• 6
• 7 The correct answer is B. This young girl presents with low weight, fear of becoming fat, and 28% of students got this correct.

• 8 amenorrhea, which are consistent with a diagnosis of anorexia nervosa. She denies binge
Send us your feedback at:
9 eating or purging behaviors, so she meets the criteria for the restricting subtype of the illness.
• medFeedback@kaplan.com
She has lost a significant amount of weight and has amenorrhea for 3 months. Her vital signs,
• 10 Please include QID m001061
physical examination, and blood work are within normal limits. It is appropriate for her to start a
• 11
structured, behaviorally-based outpatient treatment to try to normalize her eating patterns and
• 12
weight.
• 13
• 14 If outpatient treatment is not successful, hospitalization (choice A) could be considered as a
• 15 next step.
• 16 Long-term psychodynamic psychotherapy (choice C) has not been shown in controlled
• 17 studies to lead to improvement in anorexia nervosa and thus would not be the most appropriate
• 18 management step in this case .
• 19
Even though individuals who have eating disorders seem to be "fearful" of food , anti-anxiety
• 20
medications such as clonazepam (choice D) have not been found to be helpful in the
• 21
treatment of the core symptoms of anorexia nervosa.
• 22
• 23 Antidepressant medications such as fluoxetine (choice E) , although helpful in other eating
• 24 disorders such as bulimia nervosa, have not been found to significantly improve eating behavior
• 25 and increase weight in low-weight patients who have anorexia nervosa.
• 26
• 1
• 2
• 3
• 4 A 3-year-old girl is being evaluated for developmental delays. Her parents report that the pregnancy was uneventful and that the

5 delivery was without complications. The girl reached all milestones normally during her first year. Length, weight, and head

circumference were all well within the normal range during the first year. During her second year, the pediatrician noticed
• 6
deceleration of head growth, and her parents noticed some weird hand movements, disinterest in play, and lack of eye contact. Her
• 7
movements became poorly coordinated. Which of the following statements is true for the patient's condition?
• 8
• 9
• 10 0 A. Some patients may have a normal or even superior intelligence
• 11
• 12 0 B. This condition is called pervasive developmental disorder, not otherwise specified

• 13 0 C. This disease is more common in males


• 14
• 15
0 0 D. Th1s d1sease occurs almost exclusively in females

• 16
• 17
• 18
Explanation Rel<aps & Refs 0 Help 0 My Questions

• 19
The correct answer is D. The child described here has the clinical features characteristic of 15% of students got this correct.
• 20
Ret! syndrome. During the first 5 months after birth, the infant has age-appropriate motor skills,
• 21 Send us your feedback at:
head circumference, growth, and social interactions. At 6 to 30 months, the child has medFeedback@kaplan. com
• 22
progressive encephalopathy with a decline in previously developed motor and social skills. There
• 23 is gross motor development delay; loss of eye contact (autistic-like behavior); deceleration in Please include QJO Q0300m
• 24 head growth, weight growth and height growth; hypotonia; and hand wringing. Associated
• 25 featu res include seizures and irregular respiratory patterns. Long-term receptive and expressive
• 26 communication and socialization abilities remain at a developmental level of less than 1 year .
• 27 Ret! syndrome occurs almost exclusively in females .
• 28
• 29
• 30 Exit
• 1
• 2
• 3
The correct answer is D. The child described here has the clinical features characteristic of 15% of students got this correct.
• 4
Rett syndrome. During the first 5 months after birth, the infant has age-appropriate motor skills,
• 5 Send us your feedback at:
head circumference, growth, and social interactions. At 6 to 30 months, the child has medFeedback@kaplan.com
• 6
progressive encephalopathy with a decline in previously developed motor and social skills. There
• 7 Please include QID Q0300m
is gross motor development delay; loss of eye contact (autistic-like behavior); deceleration in
• 8
head growth, weight growth and height growth; hypotonia; and hand wringing. Associated
• 9
features include seizures and irregular respiratory patterns. Long-term receptive and expressive
• 10 communication and socialization abilities remain at a developmental level of less than 1 year.
• 11 Rett syndrome occurs almost exclusively in females.
• 12
Choice A , that some patients may have a normal or even superior intelligence, is not the
• 13
correct answer because this statement is true for Asperger disorder, where individuals have a
• 14
limited number of intense and highly focused interests. They may actually make great
• 15
intellectual contributions, while demonstrating social insensitivity or even apparent indifference
• 16
toward loved ones. The patient in this question stem has a regression phase of deteriorating
• 17
social and motor skills and decelerating head growth, which makes it a case of Rett syndrome.
• 18
• 19 It is incorrect to say that this condition is called pervasive developmental disorder, not otherwise
• 20 specified (choice B) . This would be the answer if a child manifests a qualitative impairment in

21 the development of reciprocal social interaction and communication but does not meet the

22 criteria for other pervasive developmental disorders. The present patient clearly meets the

criteria for Rett syndrome.
• 23
• 24 The statement that this disease is more common in males (choice C) is true for autistic
• 25 disorder, which is characterized by impairments in social interactions and communication, and
• 26 by restricted repetitive and stereotyped patterns of behavior, interests, and activities. Onset is
• 27 prior to the age of 3 years. Autistic children do not show any decline in head growth or any
• 28
• 29
• 30 Exit
• 1
• 2
• 3
progressive encephalopathy with a decline in previously developed motor and social skills. There
• 4 Please include QID Q0300m
is gross motor development delay; loss of eye contact (autistic-like behavior); deceleration in
• 5
head growth, weight growth and height growth; hypotonia; and hand wringing. Associated
• 6
features include seizures and irregular respiratory patterns. Long-term receptive and expressive
• 7
communication and socialization abilities remain at a developmental level of less than 1 year.
• 8 Ret! syndrome occurs almost exclusively in females.
• 9
• 10 Choice A • that some patients may have a normal or even superior intelligence, is not the
correct answer because this statement is true for Asperger disorder, where individuals have a
• 11
limited number of intense and highly focused interests. They may actually make great
• 12
intellectual contributions, while demonstrating social insensitivity or even apparent indifference
• 13
toward loved ones. The patient in this question stem has a regression phase of deteriorating
• 14
social and motor skills and decelerating head growth, which makes it a case of Ret! syndrome.
• 15
• 16 It is incorrect to say that this condition is called pervasive developmental disorder, not otherwise
• 17 specified (choice B) . This would be the answer if a child manifests a qualitative impairment in
• 18 the development of reciprocal social interaction and communication but does not meet the

19 criteria for other pervasive developmental disorders. The present patient clearly meets the

criteria for Ret! syndrome.
• 20
• 21 The statement that this disease is more common in males (choice C) is true for autistic
• 22 disorder, which is characterized by impairments in social interactions and communication, and
• 23 by restricted repetitive and stereotyped patterns of behavior, interests. and activities. Onset is
• 24 prior to the age of 3 years. Autistic children do not show any decline in head growth or any
• 25 hand-wringing behaviors. Their motor skills are also unaffected.
• 26
• 27
• 28
• 29
• 30
• 1
• 2
• 3
A 31-year-old man is brought to the emergency department by a friend because he has been complaining that he is having auditory
• 4
hallucinations and tremors, along with associated nausea and vomiting. He feels very anxious. On mini-mental status examination, he
• 5
scores 22 of 30, and he appears to be obtunded. From which of the following substances is this patient most likely withdrawing?
• 6
• 7
• 8 (I 0 A. Alprazolam
• 9
• 10 0 B. Caffeine
• 11 0 C. Cocaine
• 12
0 D. Heroin
• 13
• 14 0 E. Nicotine
• 15
• 16
• 17 Explanation ReKaps & Refs 0 Help 0 My Questions
• 18
• 19 The correct answer is A. This patient has the symptoms of withdrawal from benzodiazepines, 73% of students got this correct

• 20 such as alprazolam. Symptoms include insomnia, tremor, Gl distress, hallucinosis, and anxiety.
Send us your feedback at:
• 21 Withdrawal from benzodiazepines can also be accompanied by generalized seizures. medFeedback@kaplan. com
• 22 Caffeine (choice B) withdrawal does not have associated obtundation and usually manifests as Please include QIO Q0617
• 23 mild psychomotor agitation.
• 24
25 Cocaine withdrawal (choice C) is characterized by dysphoria, psychomotor agitation or

retardation , and marked fatigue.
• 26
• 27 Heroin withdrawal (c hoice D) is characterized by generalized pain, nausea, vomiting,
• 28

Exit
• 1
• 2
• 3
• 4 0 C. Cocaine
• 5
0 D. Heroin
• 6
• 7 0 E. Nicotine
• 8
• 9
• 10 Explanation ReKaps & Refs 6 Help 0 My Questions
• 11
• 12 The correct answer is A. This patient has the symptoms of withdrawal from benzodiazepines, 73% of students got this correct.

• 13 such as alprazolam. Symptoms include insomnia, tremor, Gl distress, hallucinosis, and anxiety.
Send us y our feedback at:
• 14 Withdrawal from benzodiazepines can also be accompanied by generalized seizures. medFeedback@kaplan.com
• 15 Caffeine (choice B) withdrawal does not have associated obtundation and usually manifests as Please include QID Q06 17
• 16 mild psychomotor agitation.
• 17
Cocaine withdrawal (choice C) is characterized by dysphoria, psychomotor agitation or
• 18
retardation , and marked fatigue.
• 19
• 20 Heroin withdrawal (choice D) is characterized by generalized pain, nausea, vom iting,
• 21 diarrhea, and piloerection (goose flesh)
• 22 Nicotine withdrawal (choice E) is characterized by intense craving activity and mild to
• 23 moderate psychomotor agitation.
• 24
• 25 Reviewed on 12/10112
• 26
• 27
• 28
• 2
• 3
• 4 A 25-year-old law school student comes to the physician for a follow-up visit 2 weeks after discharge from the psychiatric inpatient
• 5 unit where she was diagnosed and treated for her first episode of schizophrenia. She is compliant with her medication (olanzapine)
• 6 and seems to be tolerating it well. She no longer seems to be having psychotic symptoms, but she also is not socializing with friends or
• 7 relatives and is not interested in returning to school. She broke up with her boyfriend approximately a week before this appointment.
• 8 Her parents are concerned that she may have a hard time dealing with her illness. Which of the following is the most appropriate next
• 9 step in management?
• 10
• 11
• 12 (I 0 A. Address the pallenrs feelings and suicide risk

• 13 0 B. Check the patienrs TSH level


• 14
0 C. Discuss the doctor-patient relationship
• 15
• 16 0 D. Ensure compliance with medication
• 17
0 E. Provide education about disease and medication
• 18
• 19 0 F. Suggest that the patient goes back to school and resumes daily activities
• 20
• 21
• 22 Explanation Rel<aps & Refs 8 Help Q My Questions

• 23
• 24 The correct answer is A. Assessing potential suicidal risk is the most important goal of this 50% of students got this correct.

• 25 meeting, given the history provided by this patient's parents. In first-episode schizophrenic
Send us your feedback a t:
• 26 patients who have been successful before illness, suicide risk increases after the discharge and
medFeedback@kaplan.com
• 27 during the stabilization of psychotic symptoms. They usually feel depressed and hopeless about
Please include OlD s2s144m
the future. In this period of post-psychotic depression, suicidal risk increases and needs to be
• 28
• 29
• 30
Exit
• 31.
• 2
• 3
• 4 Rel<aps & Refs

• 5
• 6 The correct answer is A. Assessing potential suicidal risk is the most important goal of this 50% of students got this correct

• 7 meeting, given the history provided by this patient's parents. In first-episode schizophrenic
Send us your feedback at:
• 8 patients who have been successful before illness, suicide risk increases after the discharge and
medFeedback@kaplan.com
9 during the stabilization of psychotic symptoms. They usually feel depressed and hopeless about
• Please include QID s2sl44m
the future. In this period of post-psychotic depression, suicidal risk increases and needs to be
• 10
assessed.
• 11
• 12 The patient's TSH needs to be checked (choice B) because it could be a manifestation of
• 13 hypothyroidism. but not immediately. Assessing the potential risk for suicide is more important
• 14 than finding the cause of the condition.
• 15 Discussing the doctor-patient relationship (choice C) should be a part of any outpatient
• 16 session, especially establishing trust Part of it involves talking openly about issues of concern,
• 17 such as depression and suicidal ideation .
• 18
Ensuring compliance with medication (choice D) should be a part of every meeting. Part of
• 19
this is done through establishing a sense of trust and a good doctor-patient relationship. In this
• 20
case it is important but not a priority. given the case history
• 21
• 22 Providing education (choice E) about illness and medication to the patient and family
• 23 members ensures compliance and improves coping skills, and should take place on an ongoing
• 24 basis. In this case, however, the most important and acute concerns need to be addressed first
• 25
Open suggestions about resuming activities (choice F) are appropriate for patients who need
• 26
more structure and supervision. In this case it would be more appropriate to discuss the
• 27 patient's reasons for dropping out of school and breaking up with her boyfriend.
• 28
• 29
• 30
Exit
• 31.
• 3
• 4
• 5 A successful, 27 -year-old businesswoman has developed a fear of flying after an extremely rough landing. She is paralyzed with fear
• 6 and unable to travel for business. Her physician tried giving her lorazepam to take during the flight, but it didn't help. She returns to the
• 7 physician and asks ifthere is anything else that she can do to reduce her fear because she is not getting a promotion at work due to
• 8 her inability to travel. Which of the following is the most commonly used treatment for this disorder?
• 9
• 10
11 0 A. Aversive conditioning

• 12 (I 0 B. Exposure therapy
• 13
0 C. Fluoxetine
• 14
• 15 0 D. Propranolol
• 16
0 E. Supportive therapy
• 17
• 18
• 19
Explanation ReKaps & Refs G Help Q My Questions
• 20
• 21
The correct answer is B . Exposure therapy, a type of behavior therapy, is the most commonly 74% of students got this correct.
• 22
used treatment of specific phobia. The therapist either treats the condition by forced exposure to
• 23 the feared objecVevent (called flooding) or desensitizes the patient by a gradual exposure to the
Send us your feedback at:
medFeedback@kaplan.com
• 24 phobic stimulus (systemic desensitization). Relaxation and breathing control are important parts
• 25 of systemic desensitization. Please include QJO Q0468m

• 26
Aversive conditioning (c hoice A) is a term that describes the pairing of an unpleasant stimulus
• 27
with the stimulus that causes maladaptive behavior (for example, being terribly sick due to the
• 28
effects of disulfiram after drinking alcohol), so that the behavior eventually comes to elicit
• 29
• 30
Exit
• 3
• 4
• 5
• 6
• 7 Explanation ReKaps & Refs @ Help Q My Questions
• 8
• 9 The correct answer i s B. Exposure therapy, a type of behavior therapy, is the most commonly 74% of students got this correcl
• 10 used treatment of specific phobia. The therapist either treats the condition by forced exposure to
• 11 Send us your feedback at:
the feared objecUevent (called flooding) or desensitizes the patient by a gradual exposure to the
medFeedback@kaplan.com
• 12 phobic stimulus (systemic desensitization). Relaxation and breathing control are important parts
Please include QID Q0468m
• 13 of systemic desensitization.
• 14
Aversive conditioning (choice A) is a term that describes the pairing of an unpleasant stimulus
• 15
with the stimulus that causes maladaptive behavior (for example, being terribly sick due to the
• 16
effects of disulfiram after drinking alcohol), so that the behavior eventually comes to elicit
• 17
unpleasant rather than pleasant feelings , and thus eventually disappears.
• 18
• 19 Fluoxetine (choice C) has limited use in the treatment of specific phobia since behavioral
therapy is the main route of intervention. SSRis may be used as adjunctive therapy, as patients
• 20
may have coexisting anxiety disorders. So exposure therapy is a better option in this question.
• 21
• 22 Propranolol (choice D) is used in the treatment of a social phobia to block the autonomic
• 23 response, thus preventing symptoms such as tremor and palpitations, leading to successful
• 24 performance in social situations despite anxiety.
• 25 Supportive therapy (choice E) may be used in helping the patient actively confront the phobic
• 26 stimulus during treatment. It is usually used in addition to an ongoing treatment.
• 27
• 28
• 29
• 30
• 4
• 5
• 6 A mother brings in her 9-year-old son, who has presented several symptoms since the age of 6. The mother states that the boy never
• 7 pays attention to his teachers or parents, and always has trouble because of his inability to stay quiet and still. His school's principal
• 8 suspended him last month after finding him fighting and trying to destroy school property. Last week he was referred to the
• 9 psychologist for counseling and management of a learning disorder. The mother is also concerned that her 4-year-old son might also
• 10 have the same disorder, as he is having difficulty interacting with other children at the local day care, is unable to sit through cartoons

• 11 and meals, is increasingly irritable, and was late in toilet training. She and her husband are also considering having another child.

12
Which of the following should the physician advise this mother concerning her child's disorder?

• 13
• 14
0 A. It is a benign disorder and is self-limited to childhood
• 15
• 16 0 B. The child with the disorder has a 50% risk of developing antisocial personality disorder
• 17 0 C. The child with the disorder has an increased risk of developing schizophrenia, disorganized type
• 18
19 0 D. The level of the child's inattention will lessen during childhood and adolescence

• 20 (I 0 E. The older her children get, the less likely they will be to develop the disorder, as 1ts prevalence is estimated to decline as
• 21 people age
• 22
• 23
• 24 Explanation ReKaps & Refs G Help Q My Questions
• 25
• 26 The correct answer is E. Th is child's disorder is attention deficit hyperactivity disorder 54% of students got th is correct.
• 27 (ADHD). It is characterized by inattention, hyperactivity, and impulsivity that interfere with social
Send us your feedback a t:
• 28 or academic function . The symptoms last for 6 months, and onset occurs before 7 years of age. medFeedback@kaplan.com
• 29 Symptoms are present in multiple settings. Accord ing to follow-up studies, the prevalence of
Please includf' r\.,, ,....,.,. . . . ..,.,.._
• 30 AOHO i~ A~tim::otAti In t1A,-,IinA hv 'i0% AVArv 'i VA::Or~ until thA mit1-?~ ThArAfnrA it i~

• 31
• 32
Exit
• 33
• 4
• 5
• 6 0 E. The older her children get, the less likely they will be to develop the disorder, as 1ts prevalence is estimated to decline as
• 7 people age
• 8
• 9
• 10 Explanation ReKaps & Refs @ Help 0 My Questions
• 11
• 12 The correct answer i s E. This child's disorder is attention deficit hyperactivity disorder 54% of students got this correct
• 13 (ADHD). It is characterized by inattention, hyperactivity, and impulsivity that interfere with social
Send us your feedback at:
• 14 or academic function. The symptoms last for 6 months, and onset occurs before 7 years of age. medFeedback@kaplan.com
• 15 Symptoms are present in multiple settings. According to follow-up studies, the prevalence of
Please include QID Q0274m
• 16 ADHD is estimated to decline by 50% every 5 y ears until the mid-20s. Therefore, it is
• 17 acceptable to counsel this mother that the older her children get without symptoms of ADHD, the
• 18 less likely they are to have the disorder.
• 19 Choice A is incorrect because ADHD persists into adulthood in approximately 30% of affected
• 20 individuals.
• 21
Choice B is incorrect because children with ADHD have a 25% chance of developing
• 22
antisocial personality disorder.
• 23
• 24 Choice c is incorrect because there is no known association between schizophrenia and
• 25 ADHD , and no increased risk for schizophrenia beyond that of the general population.
• 26 Choic,e D is incorrect because the child's impulsivity and hyperactivity will decrease as he
• 27 grows older, but his level of inattention will not.
• 28
• 29
• 30
• 31
• 32
• 33
• 5
• 6
• 7 A patient is talking to his psychiatrist about a conflict he has with his partner. They argue because the partner complains that the
• 8 patient is inefficient and procrastinates doing things that are his responsibility. He never completes tasks in the manner in which he
• 9 was asked, but rather in his own way and usually much later. Which of the following types of defense mechanism is this patient
• 10 exhibiting?
• 11
• 12
• 13 0 A. Acting out
• 14 0 B. Blocking
• 15
• 16 (I 0 C. Passive-aggressrveness

• 17 0 0 . Regression
• 18
0 E. Splitting
• 19
• 20
• 21
• 22
Explanation ReKaps & Refs 0 Help 0 My Questions

• 23
The corr ect answer is C. With passive-aggressiveness, aggression toward others is 55% of students got this correct.
• 24
expressed indirectly, usually through procrastination, stubbornness, passivity, and forgetfulness.
• 25 Send us your feedback at:
People employing this defense mechanism might take an unnecessarily long time to get ready
• 26 medFeedback@kaplan.com
for a party they do not wish to attend. Or a person may leave notes to avoid face-to-face
• 27 Please include QJO Q0475
discussions or confrontations. The manifestations usually affect others more than the person
• 28
who is doing it.
• 29
• 30 Acting out (choice A) is a defense by which a person expresses an unconscious wish through
• 31 action to avoid bein!l conscious of the stronQ accompanyinQ affect.



Exit
• 5
• 6
• 7
• 8
• 9
• 10 Explanation Rel<aps & Refs f} Help 0 My Questions

• 11
The co rrect answer i s C. With passive-aggressiveness, aggression toward others is 55% of students got this correct
• 12
expressed indirectly, usually through procrastination, stubbornness, passivity, and forgetfulness.
• 13 Send us your feedback at:
People employing this defense mechanism might take an unnecessarily long time to get ready
• 14 medFeedback@kaplan.com
for a party they do not wish to attend. Or a person may leave notes to avoid face-to-face
• 15 Please include QID Q0475
discussions or confrontations. The manifestations usually affect others more than the person
• 16
who is doing it.
• 17
• 18 Acting out (choice A) is a defense by which a person expresses an unconscious wish through
• 19 action to avoid being conscious of the strong accompanying affect.

• 20 Blocking (choice B) is a transient inhibition ofthinking that usually involves strong affects and
• 21 impulses that are being inhibited .
• 22
Reg ression (choice D) is a defense mechanism in which a person returns to an earlier stage
• 23
of development to avoid tension at the present level of development.
• 24
• 25 Splitting (choice E) is a defense mechanism by which external objects are divided into "all
• 26 good" or "all bad". It is accompanied by abrupt shifting of an object from one category to the
• 27 other.
• 28 Reviewed on 12/10112
• 29
• 30
• 6
• 7
• 8 A 49-year-old man comes to a psychiatrist for depressive symptoms. He talks about his past psychiatric problems. He also mentions
• 9 that he is an alcoholic and has been taking disulfiram for some time to keep sober. He once tried to drink after taking the drug and
• 10 ended up being terribly sick. Which of the following principles best describes this treatment of alcoholism?
• 11
• 12
• 13 0 0 A. Cond1t1oned avoidance
• 14 0 B. Extinction
• 15
• 16 0 C. Flooding

• 17 0 D. Positive reinforcement
• 18
0 E. Reciprocal inhibition
• 19
• 20
• 21
• 22
Explanation ReKaps & Refs @ Help Q My Questions

• 23
The correct answer is A. Conditioned avoidance (or aversive conditioning) is a term that 85% of students got this correct.
• 24
describes the pairing of an unpleasant stimulus (being terribly sick due to effects of disulfiram)
• 25 Send us your feedback at:
with the stimulus that causes maladaptive behavior (alcoholism) , so that the behavior eventually
26 medFeedback@kaplan. com
• comes to elicit unpleasant rather than pleasant feelings. Another example of conditioned
• 27 Please include QIO Q0470m
avoidance is the use of chili peppers to stop thumb-sucking in children.
• 28
• 29 Extinction (choice B) requires the removal of the positive reinforcement for inappropriate
• 30 behavior so that maladaptive behavior decreases in frequency. It is often used in child
psychiatry. For example, a child is ignored (removal of positive reinforcement) by the parents
• 31
when he throws temper tantrums (maladaptive behavior). This will lead to extinction of the
• 32
• 33
• 34
Exit
• 35
• 6
• 7
• 8
• 9
The corr ect answer is A. Conditioned avoidance (or aversive conditioning) is a term that 85% of students got th is correct
• 10
describes the pairing of an unpleasant stimulus (being terribly sick due to effects of disulfiram)
• 11 Send us your feedback a t:
with the stimulus that causes maladaptive behavior (alcoholism), so that the behavior eventually medFeedback@kaplan.com
• 12
comes to elicit unpleasant rather than pleasant feelings. Another example of conditioned
• 13 Please include QID Q0470m
avoidance is the use of chili peppers to stop thumb-sucking in children.
• 14
• 15 Extinction (choice B) requires the removal of the positive reinforcement for inappropriate
• 16 behavior so that maladaptive behavior decreases in frequency. It is often used in child
• 17 psychiatry. For example, a child is ignored (removal of positive reinforcement) by the parents
when he throws temper tantrums (maladaptive behavior). This will lead to extinction of the
• 18
behavior.
• 19
• 20 Flooding (choice C) is a therapeutic technique in which a patient is exposed to the feared
• 21 situation without the possibility of escape. This experience is stressful and it must be done in a
• 22 supervised and controlled manner. For example, fear of heights is treated by having a patient
• 23 ride up an elevator.
• 24 Positive reinforcement (choice D) happens when a subject is rewarded for manifesting
• 25 desired behavior. For example, good work from an employee is rewarded by increased pay,
• 26 which encourages her to continue working well.
• 27
Reciprocal inhibition (choice E) happens when a response that is antagonistic to the
• 28
undesired behavior is paired with the behavioral response (e.g., relaxing along with
• 29
anxiety-provoking stimuli).
• 30
• 31
• 32
• 33
• 34
• 35
• 7
• 8
• 9
A 26-year-old graduate student feels depressed, suicidal, and tense, and admits to "hurting herself' for many years without the
• 10
intention to die. Rather, she explains that usually it is therapeutic to feel the pain she innicts on herself, that it helps her release
• 11
tension. She has several scars on her arms that were caused by superficial cuts made with a razor blade. She has two new superficial
• 12
cuts from the night before. She believes that the current "crisis" is related to a recent breakup with a man. She has had several stormy
• 13 relationships with men that typically end poorly. She is usually unhappy when she is alone. however, so she goes directly into another
• 14 relationship. Often she feels empty and bored, yet does not have the initiative to change things about herself. She frequently uses
• 15 drugs to elevate her mood. She attends her classes regularly and usually is able to prepare for examinations. Which of the following
• 16 is a comorbid problem of this condition?
• 17
• 18
• 19 0 A. Adjustment disorder
• 20 0 B. Dependent personality disorder
• 21
• 22 (I 0 C. Eating disorder
• 23 0 D. Narcolepsy
• 24
0 E. Narcissistic personality disorder
• 25
• 26
• 27
• 28
Explanation Rel<aps & Refs 8 Help Q My Questions

• 29
The correct answer is C. Borderline personality disorder is a pervasive pattern of instability of 33% of students got this correct.
• 30
interpersonal relationships, affects, or self-image beginning in early adulthood. Efforts to avoid
• 31 Send us your feedback at:
abandonment, a pattern of unstable relationships alternating between idealization and medFeedback@kaplan.com
• 32
• 33
devaluation, an unstable self-image, impulsivity, recurrent suicidal behavior, chronic feelings of ,...., ____ ; __... ~ .


Exit
• 7
• 8
• 9
0 B. Dependent personality disorder
• 10
• 11 f) 0 C. Eatmg disorder
• 12
0 D. Narcolepsy
• 13
• 14 0 E. Narcissistic personality disorder
• 15
• 16
• 17 Explanation ReKaps & Refs G Help 0 My Questions

• 18
• 19 The correct answer is C. Borderline personality disorder is a pervasive pattern of instability of 33% of students got this correct.

• 20 interpersonal relationships, affects, or self-image beginning in ear1y adulthood. Efforts to avoid


Send us your feedback at:
21 abandonment, a pattern of unstable relationships alternating between idealization and
• medF eedback@kaplan. com
devaluation, an unstable self-image , impulsivity, recurrent suicidal behavior, chronic feelings of
• 22
Please include QID s2s 168m
emptiness, inappropriate intense anger, transient paranoid ideation, and affective instability are
• 23
typical symptoms of this disorder. It is associated with other comorbid conditions, such as mood
• 24
disorders, anxiety, eating disorders, and substance abuse. About 50% of patients with
• 25
borderline personality disorder have eating disorders.
• 26
• 27 Adjustment disorder (choice A) • dependent personality disorder (choice B) • and narcolepsy
• 28 (choice D) • and narcissistic personality disorder (choice E) . are not associated with
• 29 borderline personality disorder.
• 30 Reviewed on 12/10112
• 31
• 32
• 33

• 8
• 9
• 10 A 34-year-old man is brought to the psychiatrist by a friend because he is very loud and talkative. He states that he has not slept for
• 11 the past 2 days because he has been working to establish a catering service, for which he has been buying many kitchen utensils,
• 12 baking supplies, and recipe books. He has also been "sampling wines from around the world" so that he can become a wine expert for
• 13 his new business. The patient's vitals are normal. Lab values show the following :
• 14 Hgb 14.2 g/dl
• 15 WBC 8,700/mm 3
• 16
Platelets 320,000/mm 3
• 17
• 18 Sodium 138mEq/L
Potassium 4.4 mEq/L
• 19
Chloride 102 mEq/L
• 20
Bicarbonate 26 mEq/L
• 21
BUN 40 mg/dl
• 22 Creatinine 2.5 mg/dl
• 23 Glucose 80 mg/dl
• 24 Calcium 9.6 mg/dl
• 25
Which of the following is the most appropriate treatment fo r this man's condition?
• 26
• 27
• 28 0 A. Benzodiazepines
• 29
0 B. Carbamazepine
• 30
• 31 0 C. Lithium
• 32
f) 0 D. Valproate
• 33
• 34
• 35
• 36
• ~
• 8
• 9
• 10
• 11 Calcium

• 12 Which of the following is the most appropriate treatment for this man's condition?
• 13
• 14
• 15 0 A. Benzodiazepines
• 16 0 B. Carbamazepine
• 17
• 18
0 C. lithium

• 19 fl 0 0 . Valproate
• 20
• 21
• 22 Explanation ReKaps & Refs fi Help 0 My Questions
• 23
Add this question to
• 24 The correct answer is D. This patient has bipolar disorder, for which the treatment of choice 60% of stude My Questions
• 25 is lithium. But this man also has renal failure , as his BUN and creatinine are high. In cases
Send us your feedback at:
• 26 where lithium is contraindicated, valproate is the drug of choice . medFeedback@kaplan. com
• 27
Benzodiazepines (choice A) are not mood stabilizers; they are used in anxiety disorders. Please include QID Q0297m
• 28
• 29 Carbamazepine (choice B) is used in the treatment of bipolar disorder when both lithium and
• 30 valproate are contraindicated.
• 31 Lithium (choice C) would be the drug of choice if the patient did not have the renal problem.
• 32
• 33
• 34
• 35
• 36
• 71
• 9
• 10
• 11 A 32-year-old African American man with a history of chronic paranoid schizophrenia is brought to his psychiatrist after fa iling
• 12 courses of treatment with both haloperidol and olanzapine. His psychiatrist would like to start him on a new antipsychotic medication .
• 13 The patient says that last month he was taken to the ER for fainting and an EKG was done. He does not remember what the diagnosis
• 14 was. Which of the following medications should the psychiatrist avoid?
• 15
• 16
• 17 0 A. Aripiprazole
• 18 0 B. Clozapine
• 19
• 20
0 C. Quetiapine

• 21 0 D. Risperidone
• 22
(I 0 E. Z1pras1done
• 23
• 24
• 25
• 26
Explanation Rel<aps & Refs @ Help 0 My Questions

• 27
The correct answer is E. Of all the indicators listed, ziprasidone causes QT prolongation. With 75% of students got this correct.
• 28
this patient's history of a fainting episode and having an EKG, arrhythmia is strongly suspected.
• 29 Send us your feedback at:
So ziprasidone should be avoided. medFeedback@kaplan.com
• 30
• 31 Aripiprazole (choice A) is not associated with any arrhythmia so is not contraindicated in this Please include QIO Q0269m
• 32 patient.
• 33 Clozapine (c hoice B) can cause neutropenia and myocarditis. It has no association with
• 34 arrhythmia, so need not be avoided .
• 35
• 36
Exit
• 9
• 10
• 11
0 B. Clozapine
• 12
• 13 0 C. Quetiapine
• 14
0 D. Risperidone
• 15
• 16 0 0 E. Zlprastdone
• 17
• 18
• 19 Explanation ReKaps & Refs @ Help 0 My Questions

• 20
• 21 The correct answer is E. Of all the indicators listed, ziprasidone causes QT prolongation. W ith 75% of students got this correct.

• 22 this patient's history of a fainting episode and having an EKG, arrhythmia is strongly suspected.
Send us your feedback at:
• 23 So ziprasidone should be avoided. medF eedback@kaplan . com
• 24 Aripiprazole (choice A} is not associated with any arrhythmia so is not contraindicated in this Please include QID Q0269m
• 25 patient.
• 26
Clozapine (choice B) can cause neutropenia and myocarditis. It has no association with
• 27
arrhythmia, so need not be avoided.
• 28
• 29 Quetiapine (choice C) is associated with the least risk of movement disorder; so whenever
• 30 movement disorder is an issue (e.g. , a patient with Huntington disease), quetiapine is preferred.
• 31 Risperidone (choice D) is not associated with any arrhythmia so need not be avoided. Among
• 32 the atypical antipsychotics, it is most strongly associated with movement disorders.
• 33
• 10
• 11
• 12 A 40-year-old man was brought in for evaluation by the Coast Guard just after the small plane he was piloting crashed into the ocean .
• 13 The man's wife and two friends were also on the plane. The man has survived the crash with cuts and a broken arm, but he claims he
• 14 has no idea how the crash happened or how he escaped the plane. He is also unable to explain how he got his life jacket on. His
• 15 physical examination is significant only for minor lacerations and a fractured right humerus, and he has no alteration in
• 16 consciousness. A CT scan is normal. He is very upset that his wife's fate is unknown, and he has nightmares for the next several

• 17 nights while trying to sleep. Which of the following is the most likely diagnosis?

• 18
• 19
(I 0 A. Dtssoctatrve amnesia
• 20
• 21 0 B. Dissociative fugue
• 22 0 C. Dissociative identity disorder
• 23
• 24 0 D. Factitious disorder
• 25 0 E. Post-traumatic stress disorder
• 26
• 27
• 28 Explanation ReKaps & Refs @ Help 0 My Questions
• 29
• 30 The correct answer is A. In this instance, an extremely stressful event has been followed by 60% of students got this correct.
• 31 localized loss of memory or amnesia of circumstances surrounding the event, making
32 Send us your feedback at:
• dissociative amnesia the most likely diagnosis. Dissociative amnesia is often accompanied by medFeedback@kaplan.com
• 33 nightmares and anxiety concerning the event, both of which this patient also has.
Please include QID Q0397m
• 34
Dissociative fugue (choice B) is a disturbance of identity that requires sudden, unexpected
• 35
travel away from home or one's place of work, with inability to recall one's past. The factthatthis
• 36
• 37
• 38
Exit
• 39
• 10
• 11
• 12
• 13 The correct answer is A. In this instance, an extremely stressful event has been followed by 60% of students got this correct

• 14 localized loss of memory or amnesia of circumstances surrounding the event, making


Send us your feedback at:
• 15 dissociative amnesia the most likely diagnosis. Dissociative amnesia is often accompanied by medFeedback@kaplan.com
• 16 nightmares and anxiety concerning the event, both of which this patient also has.
Please include QIO Q0397m
• 17 Dissociative fugue (choice B) is a disturbance of identity that requires sudden, unexpected
• 18 travel away from home or one's place of work, with inability to recall one's past. The fact that this
• 19 patient was piloting a small plane does not point toward unexpected travel.
• 20
Dissociative identity disorder (choice C) is also a disturbance of identity. It requires the
• 21
presence of two or more distinct identities or personality states, which recurrenUy take control
• 22
of the person's behavior. The fragmenting of a distinct and integrated personality into multiple
• 23
personalities may have the adaptive purpose of protecting the more vulnerable and frightened
• 24 facets of the core personality after trauma. This is popular1y known as multiple personality
• 25 disorder.
• 26
27 Factitious disorder (choice D) is a diagnosis requiring intentional production of symptoms and

gratification from assuming the sick role.
• 28
• 29 Post-traumatic stress disorder (PTSD; choice E ) is the diagnosis when the patient has (for
• 30 more than 1 month) symptoms of:
• 31
• Re-experiencing the traumatic event in the form of nightmares, flashbacks, etc.
• 32
• Avoidance of the stimuli associated with the trauma (e.g., for this patient, avoiding going
• 33 near an airplane or an ocean beach)
• 34 • Anxiety, sleep disturbances, startle reflex
• 35
The patient in this question does not fulfill the criteria for PTSD. Moreover, his symptoms just
• 36
• 37
• 38
Exit
• 39
• 10
• 11
• 12 medFeedback@kaplan.com
nightmares and anxiety concerning the event, both of which this patient also has.
• 13 Please include QID Q0397m
• 14 Dissociative fugue (choice B ) is a disturbance of identity that requires sudden, unexpected
• 15 travel away from home or one's place of work, with inability to recall one's past. The fact that this
• 16 patient was piloting a small plane does not point toward unexpected travel.
• 17 Dissociative identity disorder (c hoice C ) is also a disturbance of identity. It requires the
• 18 presence of two or more distinct identities or personality states, which recurrenUy take control
• 19 of the person's behavior. The fragmenting of a distinct and integrated personality into multiple
• 20 personalities may have the adaptive purpose of protecting the more vulnerable and frightened
• 21 facets of the core personality after trauma. This is popularly known as multiple personality
• 22 disorder.
• 23
Factitious disorder (choice D) is a diagnosis requiring intentional production of symptoms and
• 24
gratification from assuming the sick role.
• 25
• 26 Post-traumatic stress disorder (PTSD; choice E ) is the diagnosis when the patient has (for

• 27 more than 1 month) symptoms of:

• 28 • Re-experiencing the traumatic event in the fo rm of nightmares, flashbacks, etc .


• 29 • Avoidance of the stimuli associated with the trauma (e. g .. for this patient, avoiding going
• 30 near an airplane or an ocean beach)
• 31 • Anxiety, sleep disturbances, startle reflex
• 32
The patient in this question does not futfill the criteria for PTSD. Moreover, his symptoms just
• 33
started. PTSD cannot be diagnosed until 1 month or more after of the event.
• 34
• 35
• 36
• 37
• 38
• 39
• 11
• 12
• 13 A 24-year-old male college freshman presents to his physician for a routine checkup during winter break after his first week of finals .
• 14 He reports no problems. On a routine psychiatric screening, however, the patient discloses that this past week has been "crazy." He
• 15 begins by describing how "dangerously fast" he has been driving. When asked to be more specific, the patient apologizes for not
• 16 hearing you and comments on the paintings in the examination room. He admits that his attention has been lacking because he is
• 17 ''thinking so much." Still, he says that he has made frequent trips to the library and has been able to "stay up all night to study without a
• 18 problem," which he feels compensates for his inattention during the last week of lectures. He states confidently that he probably
• 19 "aced" each examination. He digresses to describe the intricate architecture of the library, confused about why he only recently
• 20 noti ced it. On physical examination, his pupils are noted to be dilated. Which of the following is the most likely diagnosis?

• 21
• 22
0 A. Adult-onset attention deficiVhyperactivity disorder (ADHD)
• 23
• 24 0 B. Bipolar I disorder, single manic episode
• 25
0 C. Bipolar II disorder
• 26
• 27 (I 0 D. Cocame abuse
• 28 0 E. Cyclothymic disorder
• 29
30
0 F. Delusional disorder

• 31
• 32
• 33 Explanation ReKaps & Refs 6 Help 0 My Questions

• 34
The correct answer is D. The diagnosis is cocaine abuse because that is the only option that 71 % of students got this correct.
• 35
can cause manic symptoms together with dilated pupils.
• 36 Send us your feedback at:
• ~ ; ............ ........ ............................. .,. ,...,..,, .. ,.. ............ I\ nun 1-a..-=-- "'' . . . . . . . . . . . . '"';....... ,,..,;,. . . . . . . Aa..... ........
~-.. t..a ........ ;,.. ;,. m.otl~iOjOflh;,rk



Exit
• 11
• 12
• 13
• 14 Explanation ReKaps & Refs 6 Help Q My Questions
• 15
• 16 The correct answer is D. The diagnosis is cocaine abuse because that is the only option that 71% of students got this correct.

• 17 can cause manic symptoms together with dilated pupils.


Send us your feedback at:
• 18 This man does not have adult-onset ADHD (choice A) because his pupils are dilated. Mania is medFeedback@kaplan.com
• 19 distinguished from ADHD by its later onset, decreased need for sleep, acute nature, and Please include QID mOOOS37m
• 20 elevated mood.
• 21
Bipolar I disorder, single manic episode {choice B) , would be the correct answer if his pupils
• 22
were not dilated. This patient has all symptoms of BPD 1: decreased need for sleep, increase in
• 23
goal-directed activity, distractibility, inflated self-esteem, and racing thoughts. The papillary
• 24
finding makes this a wrong choice.
• 25
• 26 Bipolar II disorder (choice C) is characterized by the presence or history of a depressive
• 27 episode in combination with the presence or history of a hypomanic episode. The patient's
• 28 mania is significant enough to be considered a full manic episode, not hypomania.
• 29 Cyclothymic disorder (choice E) is comprised of a history of numerous hypomanic episodes
• 30 and episodes of depressive symptoms, none of which meet the criteria for a major depressive
• 31 episode .
• 32
Delusional disorder (choice F) is incorrect. Although this man believes that he "aced" his
• 33
exams, he does not meet the criteria for delusional disorder. Additionally, delusional disorder
• 34
does not explain his other symptoms.
• 35
• 36
• 12
• 13
• 14 A 63-year-old white man who recently retired after working as a plumber for over 30 years returns to his family physician, saying that
• 15 he has been feeling ''very down" lately and has been having decreased appetite and a loss of interest in activities that used to give him
• 16 pleasure. He is a smoker, drinks no alcohol, and is being treated by his family physician for moderate essential hypertension. His
• 17 physical examination is unchanged from his previous visits. Which of the following is the most appropriate next step in management?
• 18
• 19
• 20 0 A. Discuss actMties that will help him enjoy his retirement

• 21 0 B. Order a thyroid-stimulating hormone (TSH) level


• 22
0 C. Order electroconvulsive therapy (ECT)
• 23
• 24 0 D. Prescribe an antidepressant
• 25 (I 0 E. Revtew the patienfs medication history
• 26
• 27
• 28
Explanation Rel<aps & Refs fj Help Q My Questions
• 29
• 30
The correct answer is E. Many medications used to control hypertension, such as 34% of students got this correct.
• 31
propranolol, and in the past, reserpine, are known to occasionally lead to depressive symptoms.
• 32 By evaluating the patient's medication record , the physician can determine which medications
Send us your feedback at:
medFeedback@kaplan.com
• 33 were started at what time and can adjust dosages or switch to alternative medications to control
• 34 hypertension. Changing the antihypertensive medication will possibly improve the depressive Please include QIO Q0264m

• 35 symptoms without the need to start an antidepressant (choice D) .


• 36
Discussing activities that will help him enjoy his retirement (choice A) is an appropriate
• 37
intervention but is not the most appropriate next step, inasmuch as the etiology of the patient's
• 38
• 39
• 40
Exit
• 4 1.
• 12
• 13
• 14
• 15 f) 0 E. ReVJew the patient's medication history
• 16
• 17
• 18 Explanation ReKaps & Refs f) Help 0 My Questions
• 19
• 20 The c orrect answer i s E. Many medications used to control hypertension, such as 34% of students got this correcL
• 21 propranolol, and in the past, reserpine, are known to occasionally lead to depressive symptoms.
Send us your feedback at:
• 22 By evaluating the patienrs medication record, the physician can determine which medications
medFeedback@kaplan.com
• 23 were started at what lime and can adjust dosages or switch to alternative medications to control
Please include QID Q0264m
• 24 hypertension. Changing the antihypertensive medication will possibly improve the depressive
• 25 symptoms without the need to start an antidepressant (choice D) .
• 26 Discussing activities that will help him enjoy his retirement (choice A) is an appropriate
• 27 intervention but is not the most appropriate next step, inasmuch as the etiology of the patienrs
• 28 depression may be overlooked .
• 29
Ordering a TSH level (choice B) is also appropriate, as hypothyroidism can be an organic
• 30
cause of depression. However, given this patient's medication history, evaluating possible
• 31
pharmacologic causes of depression takes precedence in management of his depression.
• 32
• 33 Electroconvulsive therapy (choice C) is indicated in severe intractable depression when not
• 34 contraindicated by seizure disorder or other factors, but is not indicated in the present case.
• 35 Reviewed on 1216112
• 36
• 37
• 38
• 39
• 40
• 4 1.
• 13
• 14
• 15 A 36-year-old woman comes to her gynecologist because of a 3-month history of amenorrhea. Until this time, her menstrual periods
• 16 had been regular. She also complains of decreased sex drive, worsening over the past couple of months. The patient denies any other
• 17 symptoms. She has no significant medical history, although she started seeing a psychiatrist 5 months ago after a brief hospitalization
• 18 during which she was diagnosed with major depressive disorder, severe, with psychotic features. Her depressive symptoms are
• 19 resoMng. Which of the following medications is most likely responsible for the patient's presenting complaints during this visit to her
• 20 gynecologist?

• 21
• 22
0 A. Fluoxetine
• 23
• 24 0 B. Quetiapine
• 25
26
0 0 C. R1spendone

• 27 0 D. Trazodone
• 28 0 E. Venlafaxine
• 29
• 30
• 31 Explanation ReKaps & Refs @ Help 0 My Questions
• 32
• 33 The correct answer is C. Risperidone is an atypical antipsychotic agent. Conventional 50% of students got this correct.
• 34 antipsychotics are clearly associated with elevations in plasma prolactin concentrations due to
35 Send us your feedback at:
• blockade in the tuberoinfu ndibular dopaminergic pathway. Dopamine binds to pituitary medFeedback@kaplan.com
• 36 lactotrophs to inhibit the release of prolactin. Conventional antipsychotics block dopamine
Please include QID Q0857m
• 37 receptors, which releases this inhibition. The newer atypical antipsychotics have minimal, if any
• 38 effect on plasma prolactin concentrations, except for risperidone, which is associated with
• 39 P.IAv::!IAI1 nrnl:::~~tin Antin~v~hnti~-inrl11~At1 hvnP.mrnl:::~r.tinP.mi:::~ m:::~v r.:::~11~P. ~irlP. P.ffP.r.l~ in~h u1inn

• 40
Exit
• 13
• 14
• 15
The correct answer is C. Risperidone is an atypical antipsychotic agent. Conventional 50% of students got this correct.
• 16
antipsychotics are clearly associated with elevations in plasma prolactin concentrations due to
• 17 Send us your feedback at:
blockade in the tuberoinfundibular dopaminergic pathway. Dopamine binds to pituitary medFeedback@kaplan.com
• 18
lactotrophs to inhibit the release of prolactin. Conventional antipsychotics block dopamine
• 19 receptors, which releases this inhibition. The newer atypical antipsychotics have minimal, if any Please include QID Q0857m
• 20 effect on plasma prolactin concentrations, except for risperidone, which is associated with
• 21 elevated prolactin. Antipsychotic-induced hyperprolactinemia may cause side effects including
• 22 amenorrhea and infertility, sexual dysfunction, galactorrhea, and weight gain. Given this
• 23 patient's noncontributory medical history and lack of other symptoms, the onset of her
• 24 complaints correlates with the initiation of treatment for psychotic depression. A morning plasma
• 25 prolactin level should be obtained.
• 26
Fluoxetine (choice A) is a selective serotonin reuptake inhibitor (SSRI). It does not cause
• 27
significant elevations in serum prolactin. Although SSRis are commonly associated with sexual
• 28
side effects, including diminished libido but especially delayed orgasm, they are not associated
• 29 with amenorrhea.
• 30
Quetiapine (choice B) is an atypical antipsychotic agent that is not associated with significant
• 31
or persistent elevations in plasma prolactin concentrations, unlike risperidone .
• 32
• 33 Trazodone (choice D ) is an older antidepressant agent that is commonly used for insomnia
• 34 associated with depression. Although it may cause sexual side effects, it does not interfere with
• 35 menstruation.
• 36
Venlafaxine (c hoice E) is a serotonin and norepinephrine reuptake inhibitor that may cause
• 37 sexual side effects but does not cause amenorrhea.
• 38
• 14
• 15
• 16
A 48-year-old woman with a history of depression has been maintained on phenelzine for several years. She comes to the physician
• 17
complaining about the worsening of her depression and insists on switching to a newer medication, such as an SSRI. She is also tired
• 18
of slicking to such a rigid diet. Which of the following statements concerning switching her from phenelzine to an SSRI is correct?
• 19
• 20
• 21 0 A. Abrupt switching may cause hypotension
• 22
0 B. A 4-week washout after discontinuing phenelzine is needed before starting an SSRI
• 23
• 24 fl 0 C. A 14-day washout period after discontinuing phenelzine 1s needed before an SSRI can be started
• 25
0 0 . It is safe to switch right away
• 26
• 27 0 E. Stopping phenelzine causes discontinuation syndrome
• 28
• 29
• 30 Explanation ReKaps & Refs G Help 0 My Questions
• 31
• 32 The correct answer is C. Serotonin syndrome can develop with concomitant use of MAO 79% of students got this correct

• 33 inhibitors and serotonergic agents, or if there has been too short a period of washout in
Send us your feedback at:
• 34 crossing over from one to another agent. At least 10 days after MAO discontinuation and 5 medFeedback@kaplan. com
• 35 weeks after discontinuation of fluoxetine should pass to avoid possible complication. Clinical
Please include QIO s2s 159m
36 presentation includes tremor, hypertension, confusion, incoordination, fever, myoclonus,

diarrhea, restlessness, and diaphoresis.
• 37
• 38 Abrupt switching (choice A) may cause serotonin syndrome, not hypotension .
• 39
Waiting 4 weeks (choice B) after discontinuing an MAO inhibitor is not necessary. A period of
• 40
• 41
• 42
Exit
• 43
• 14
• 15
• 16
• 17 0 D. It is safe to switch right away
• 18
0 E. Stopping phenelzine causes discontinuation syndrome
• 19
• 20
• 21
• 22
Explanation ReKaps & Refs @ Help 0 My Questions

• 23
The correct answer is C. Serotonin syndrome can develop with concomitant use of MAO 79% of students got this correct.
• 24
inhibitors and serotonergic agents, or if there has been too short a period of washout in
• 25 Send us your feedback at:
crossing over from one to another agent At least 10 days after MAO discontinuation and 5 medFeedback@kaplan.com
• 26 weeks after discontinuation of fluoxetine should pass to avoid possible complication. Clinical
• 27 presentation includes tremor, hypertension, confusion, incoordination, fever, myoclonus,
Please include QID s2s159m

• 28 diarrhea, restlessness , and diaphoresis.


• 29
Abrupt switching (choice A} may cause serotonin syndrome, not hypotension.
• 30
• 31 Waiting 4 weeks (choice B) after discontinuing an MAO inhibitor is not necessary. A period of
• 32 10 to 15 days should be enough for a washout from phenelzine.
• 33
Switching right away (choice D) is not safe because of the possibility of serotonin syndrome.
• 34
• 35 Stopping phenelzine does not cause the symptoms of discontinuation (choice E) usually seen
• 36 with short-half-life SSRis.
• 37 Reviewed on 1216112
• 38
• 39
• 40
• 41
• 42
• 43
• 15
• 16
• 17 A 20-year-old man presents to a psychiatrist on referral from his parole officer. The patient was formerly arrested and incarcerated
• 18 for drug possession and armed robbery and was released on parole last week. The patient laughs about the harm he caused to his
• 19 victims, shares an inconsistent employment history, and reveals numerous apartment evictions and an ability to "always fool the
• 20 landlords." The patient also tells the physician that he does not "care who [the physician] is" and will"mess him up" if he "cuts into his
• 21 brain." In an attempt to threaten the physician, the patient proceeds to describe physical fights in which he was involved. The
• 22 psychiatrist asks how long the patient remembers having legal problems, to which the patient responds, "My whole life!" There is no
• 23 evidence for mania during the course of the interview. Which of the following would confirm the diagnosis?
• 24
• 25
0 A. Confirmation of at least 5 police arrests after the age of 18 years
• 26
• 27 f) 0 B. Evtdence of conduct disorder
• 28
0 C. Evidence of inappropriate sexual behaviors
• 29
• 30 0 D. Evidence of oppositional defiant disorder
• 31 0 E. Pattern of unstable interpersonal relationships
• 32
• 33
• 34
Explanation Rel<aps & Refs ~ Help 0 My Questions
• 35
• 36 The correct answer is B. This patient has symptoms that are classic for antisocial personality 67% of students got this correct.
• 37 disorder (ASPD), a pervasive pattern of disregard for and violation of the rights of others. It is
• 38 accompanied by at least 3 of the following: fa ilure to conform to social norms or laws as
Send us your feedback at:
medFeedback@kaplan.com
• 39 indicated by repeated arrests, deceitfulness, irritability and aggressiveness; disregard for the
• 40 Please include QID m000626m
safety of others; irresponsibility; and a lack of remorse after mistreating others. One key
• 41 ··-··--··- -·-----· =- ...... ... __ .... ___ .: __ • ._ __ - . . :_. ___
_ .~. - .C- - --' ·· -·..a ; __ _ _.. __ , __ , - "' : - -

• 42
Exit
• 15
• 16
• 17 0 C. Evidence of inappropriate sexual behaviors
• 18
• 19 0 D. Evidence of oppositional defiant disorder

• 20 0 E. Pattern of unstable interpersonal relationships


• 21
• 22
• 23 Explanation ReKaps & Refs @ Help 0 My Questions
• 24
• 25 The correct answer is B. This patient has symptoms that are classic for antisocial personality 67% of students got this correct.
• 26 disorder (ASPD). a pervasive pattern of disregard for and violation of the rights of others. It is
27 Send us y our feedback at:
• accompanied by at least 3 of the following: failure to conform to social norms or laws as medFeedback@kaplan.com
• 28 indicated by repeated arrests, deceitfulness, irritability and aggressiveness; disregard for the
Please include QID m000626m
• 29 safety of others; irresponsibility; and a lack of remorse after mistreating others. One key
• 30 unknown element is whether the patient has a history of conduct disorder (CD). CD is a
• 31 disorder of childhood that manifests before age 18 years and is considered a necessary
• 32 harbinger for the development and diagnosis of ASPD.
• 33 Although patients with ASPD often engage in unlawful behavior (choice A) , and this criteria is
• 34 included in the diagnostic criteria fo r ASPD, there is no minimum requirement for police arrests.
• 35
• 36 Oppositional defiant disorder, inappropriate sexual behaviors, and unstable relationship patterns
• 37 (choices C, D, and E) are not included in the diagnostic criteria for ASPD.
• 38 Reviewed on 12/10112
• 39
• 40

You might also like